SlideShare uma empresa Scribd logo
1 de 109
LBS1 Practice Test
for Saint Xavier University
Candidates

Dr. Candace Baker

1
02/01/14
Special Education: Teaching
Students with Learning Disabilities

Dr. Candace Baker

2
02/01/14
Q1. Students with learning disabilities are
more likely than students without disabilities
to exhibit which of the following?
A.
B.
C.
D.

Clear Dominance of the left Brain for
Learning
Deficits in long-term memory retrieval
Selective attention disorders
Characteristics of giftedness in artistic
expression

Dr. Candace Baker

3
02/01/14
Q1 -- Answer
 The

Correct Answer is C.

 Research

indicates that students with learning
disabilities are likely to lag behind other children in
their ability to identify and focus on the key aspects of
a task while disregarding the less important aspects. It
has not been proven that students with learning
disabilities are more likely than other students to have
either left-brain or right-brain dominance for learning,
and students with learning disabilities have no
particular difficulty with long-term memory retrieval
(Choice B). There is no clear evidence to suggest that
students with learning disabilities are more gifted than
others in artistic expression (Choice D).

Dr. Candace Baker

4
02/01/14
Q2. Which of the following best represents
current research findings on brain damage /
dysfunction as it relates to learning disabilities?
A.

B.

C.

D.

Direct, causal links have been found between
prenatal exposure to toxic substances and certain
well-defined learning disabilities.
Brain damage/dysfunction has been ruled out
altogether as a factor in the occurrence of learning
disabilities.
Brain damage/dysfunction caused by accident or
abuse is clearly linked to learning disabilities, but
such a link has not been established for damage
that is prenatal.
There is little direct evidence of a link between
learning disabilities and brain damage/dysfunction
but some evidence that is correlational.

Dr. Candace Baker

5
02/01/14
Q2 -- Answer
Best Answer is D.
 Specific causal linkages between pre- or
postnatal development and/or experiences
and learning disabilities, as indicated in
choice A and choice C, have not yet been
established with certainty.
Because
research is ongoing in this area, choice B
is not correct.
 The

Dr. Candace Baker

6
02/01/14
Q3.
Which of the following single
indicators is most commonly used to
signify students’ eligibility for services for
specific learning disabilities?
A.
B.
C.
D.

Academic Achievement that is hampered by
social and emotional deficits
Academic achievement that is significantly
below grade level.
Academic aptitude that is significantly lower
than the norm for the chronological age.
Academic aptitude that is significantly
higher than academic achievement.

Dr. Candace Baker

7
02/01/14
Q3 -- Answer
Best Answer is D.
 Although definitions of learning disabilities
vary, most have an aptitude-achievement
discrepancy component.
Social and
emotional deficits (choice A) are not primary
to identifying learning disabilities. Choice B
as a single indicator is incorrect; many factors
other than the presence of a learning disability
could account for low academic achievement.
Chronological age related to aptitude (choice
C) is not a primary indicator for learning
disabilities.
 The

Dr. Candace Baker

8
02/01/14
Q4. A Student with learning disabilities who reads
at grade level is enrolled in a regular biology class.
The special education teacher is trying to determine
what strategies to use to help the student perform well
in the class and pass the required tests. Which of the
following Steps should the special education teacher
take first?
A.
B.
C.
D.

Determine how well the student organizes and
retains material that is representative of the course.
Obtain the required reading materials and highlight
important concepts for the student.
Secure a tape recorder for the student to use to tape
the lectures and other teacher presentations.
Use a readability index to determine the difficulty of
the required reading materials.

Dr. Candace Baker

9
02/01/14
Q4 -- Answer
 The

correct answer is A.

 Many

students with learning disabilities have
difficulty organizing materials for study and using
strategies that aid their retention of information. As a
first step, highlighting information (choice B) would
not be useful; it would not provide information about
the student, who might not have a deficit that requires
this step. Choice C would not be useful as a first step,
before the teacher learns the student’s specific study
needs. Because the student reads at grade level it
would be more advantageous to determine how the
student approaches study tasks than to determine the
difficulty of the material (Choice D).

Dr. Candace Baker

10
02/01/14
Q5. Ralph is a fourth grade, recently mainstreamed
student who has a learning disability that causes him to
decode written materials in a slow and labored manner.
His comprehension is excellent. His teacher has organized
the classroom in five reading / discussion groups to which
students have been assigned based on their reading
performance. Ralph is placed in the highest group. Which
of the following best justifies the teacher’s decision?
A.
B.
C.
D.

Ralph will have his strengths reinforced and will likely receive
spontaneous peer tutoring.
Ralph will be forced to decode faster in order to be competitive
with the other students.
The social benefits of being in the higher group will outweigh
any instructional disadvantage Ralph experiences.
Ralph can keep pace by listening to the discussion and thereby
avoid some of the decoding tasks.

Dr. Candace Baker

11
02/01/14
Q5 -- Answer
 The

Best Answer is A.

 Because

Ralph has no trouble understanding what he
reads, he is more likely to improve his skills in the
highest group than in one in which the students have
difficulty with both decoding and comprehension.
The teacher can employ strategies that will aid him in
decoding the materials. Trying to decode faster
without any type of adaptations (choice B) will likely
prove frustrating and counterproductive for Ralph.
The teacher should no undervalue Ralph’s
instructional needs (choice C), and avoiding decoding
(choice D) would not be beneficial for Ralph.

Dr. Candace Baker

12
02/01/14
Special Education: Teaching
Students with Behavioral
Disorders/Emotional Disturbance

Dr. Candace Baker

13
02/01/14
Q1. A school is considering adopting a “get tough”
policy that enables it to expel students whose
behavior is disruptive and/or violent. Which of the
following best describes the relationship between
this school’s policy and IDEA?
A.
B.
C.

D.

It is consistent with the requirements of IDEA.
This practice probably is illegal for many students
covered by IDEA.
IDEA does not address administrative policies
concerning students who are disruptive and/or
violent.
IDEA permits the expulsion of disruptive and/or
violent students only if they lack the intellectual
capacity to do schoolwork.

Dr. Candace Baker

14
02/01/14
Q1 -- Answer
 Based

on case law, the best response is B.

 Under

EHA (PL94-142) and its amendments, the
courts have established that schools may not expel
students with disabilities without due process if the
behavior causing the expulsion proceedings is related
to their disabilities. The provisions of EHA and IDEA
that have been cited as relevant are those that call for
education in the least restrictive environment and those
related to placement decisions, with expulsion being
viewed by the courts as an unplanned change in
placement not in the student’s best educational
interests. Choices A, C, and D, therefore, are incorrect.

Dr. Candace Baker

15
02/01/14
Q2. On school party days, two third-grade students with
EBD get so excited that their behavior becomes very
difficult to manage. The third-grade general education
teacher has asked the special education teacher not to send
these students to the class on the days of school parties
because the students are so disruptive. Which of the
following would be the best way for the special education
teacher to deal with this problem?
A.
B.

C.
D.

Urge the general education teacher to make allowances for these
students’ behaviors in these kinds of situations.
Agree to allow the students with emotional / behavioral disorders to
remain in the special education classroom with a member of the
nonprofessional staff during parties.
Advise the students that they will not be allowed to go to school parties
and offer them the option to stay home from school on those days.
Help the regular education teacher arrange to have additional adults
present to help with these students during parties.

Dr. Candace Baker

16
02/01/14
Q2 -- Answer
choices except D, the best answer, allow the EBD
students to be “excluded” from important school
events with their peers. No matter how severe their
behavior has been in the past, EBD students have the
right to be included in current educational activities,
even if that means using a variety of support strategies
such as additional adult assistance. It is the EBD
teacher’s role to do more than just “try to persuade”
the mainstream or general education teacher to include
the EBD students.
It is the EBD teacher’s
responsibility to see that it is done, and to help the
EBD students sustain their involvement in the activity
for as long as they are able.

 All

Dr. Candace Baker

17
02/01/14
Q3. Which of the following is NOT an
important consideration in the process of
evaluating a student for evidence of EBD?
A.
B.
C.
D.

The referring Teacher
The age of onset of the problem
behavior.
The setting in which the problem
behavior is exhibited
The treatment to be used.

Dr. Candace Baker

18
02/01/14
Q3 -- Answer
Choices

A, B, and C are all

considerations in the evaluation process.
The correct response, D, is a part of the
educational plan that is developed based
on the completion of the evaluation
process.

Dr. Candace Baker

19
02/01/14
Q4. If a teacher aide is assigned to assist in a
classroom for students with EBD, which of the
following guidelines would best help the aide to
assist students who are doing independent work?
A.
B.
C.
D.

Give the students frequent encouragement to
get them to complete their work on their own.
Provide continuous feedback to students as
they work.
Show students how to complete their tasks,
doing some of the work for them if they delay.
Limit assistance to that help needed to keep
students working on their own.

Dr. Candace Baker

20
02/01/14
Q4 -- Answer
Independent

work periods are intended
to help students learn to use their time
well and strengthen their problemsolving skills. Students need to strike
a balance between receiving needed
assistance and coming to depend too
heavily on adult intervention; thus, the
best response is D.

Dr. Candace Baker

21
02/01/14
Q5. Maria, a student with EBD, has just been placed
in a regular mathematics class. Two days in a row,
when given a math worksheet, Maria has looked at it
for a few minutes, pushed it aside, and put her head
down on her desk. Which of the following does the
teacher need to do before giving Maria more math
worksheets in class?
A.
B.
C.
D.

Ask one of the other students to work with Maria on
one of the rejected worksheets.
Set up a bonus point system with Maria to encourage
her to do her math work.
Allow Maria to take her math work home to see if
she can complete it when not under pressure.
Talk with Maria and evaluate whether the math
assignments are appropriate for her.
22

Dr. Candace Baker

02/01/14
Q5 -- Answer
The

Best Answer is D.

The

teacher must first determine if the
math assignments are appropriate for
Maria before any of the other
strategies might be effective.

Dr. Candace Baker

23
02/01/14
Q6. Which of the following guidelines being
stated by a teacher is an example of a
dependent group contingency?
A.

B.

C.

D.

“As soon as each student makes it through an entire
home economics class without one reprimand for
disruptive behavior, we will prepare lunch for class.”
“If each student meets his or her self-monitoring goal
on Tuesday, the whole class can skip a homework
assignment.”
Each student who finishes her or his homework
during the allotted fifteen-minute homework drill will
receive a coupon for a fast-food restaurant.”
“Each time Johanna earns 10 points for completing
academic tasks, the entire class will be rewarded with
five minutes of free time.”
Dr. Candace Baker

24
02/01/14
Q6 -- Answer
 Choice

D, the correct response, fits the guidelines for

dependent group contingencies. The peer performance
of certain group members determines the consequence
received by the entire group. Choices A and B
represent an interdependent group contingency. In this
strategy, each student must reach a prescribed level of
behavior before the entire group receives a
consequence. Choice C represents an independent
group contingency.
In this variation, the same
consequence is applied to individual group members.

Dr. Candace Baker

25
02/01/14
Special Education: Application of
Core Principles Across Categories
of Disabilities

Dr. Candace Baker

26
02/01/14
Q1. A 13-year-old student with a severe degree of
mental retardation has a verbal expressive
vocabulary of a few isolated words. Which of the
following augmentative communication systems
can most immediately be used in a community
setting with the students?
A.
B.
C.
D.

Sign language
Cued Speech
A communication board using pictures
A communication board using the alphabet
and number system.

Dr. Candace Baker

27
02/01/14
Q1 -- Answer
The

best answer is C.

A

communication board using pictures is most
likely to be effective given the severity of the
student’s disability and the limited nature of the
student’s current language skills.

Dr. Candace Baker

28
02/01/14
Q2. Computer-assisted instruction has been shown
to be an important teaching method for students with
mental retardation for all of the following reasons
EXCEPT:
A.
B.

C.
D.

The computer requires little teacher intervention,
freeing the teacher to prepare upcoming lessons.
The computer can be programmed to
immediately deliver feedback on the correctness
of a response.
The computer can be programmed to provide as
much repetition as a student needs.
Computer graphics and sound can maintain a
student’s motivation and attention to task.

Dr. Candace Baker

29
02/01/14
Q2 -- Answer
correct answer is A;
 You are to choose the false statement.
Effective computer-assisted instruction
tends to require as much teacher
monitoring and intervention as other types
of individualized student work.
 The

Dr. Candace Baker

30
02/01/14
Q3. To best assure the transfer of word
attack skills to newspaper reading, a
middle school teacher of students with
mild mental retardation should…
A.
B.
C.
D.

prepare teacher-made newspaper articles for
the students to read.
select articles from the local newspaper for
students to read.
develop writing exercises using words from
newspaper articles.
prepare worksheet exercises based on single
sentences from newspaper articles.

Dr. Candace Baker

31
02/01/14
Q3 -- Answer
best answer is B,
 Since one component of successful
transfer of learning is the similarity
between the situation in which a skill is
learned and the situation to which it is to
be applied.
 The

Dr. Candace Baker

32
02/01/14
Q4. Which of the following is an example
of functional language training for
children who are nonverbal?
A.

B.
C.
D.

Rewarding the child for making a
vocalization approximating a sound made
by the teacher.
Rewarding the child for any vocalization
made while looking at the teacher.
Drilling on bilabial sounds so the student
can say “mama”.
Drilling the child on pointing to the sink
when he or she wants a drink of water.

Dr. Candace Baker

33
02/01/14
Q4 -- Answer
best answer is D,
Since functional language training
involves giving the child a means
of influencing and interacting with
the environment. Teaching the
child to make sounds or imitate
words in isolation does not fulfill
the goal of functional language
training.
The

Dr. Candace Baker

34
02/01/14
Q5. A well-made teacher-developed test is
generally preferred to a standardized
achievement test when measuring learning
mastery because it . . .
A.
B.
C.
D.

is more likely to yield a true score
has higher interrater reliability
allows greater comparison of students
to each other
has better content validity

Dr. Candace Baker

35
02/01/14
Q5 -- Answer
The

best answer is D.

 “Learning

mastery” generally refers to
those lessons taught in the classroom. A
teacher-developed test is more likely
than a standardized achievement test to
assess students’ mastery of those lessons
and, therefore, to have better content
validity.

Dr. Candace Baker

36
02/01/14
Q6. Which of the following best
describes an ecological inventory?
A.
B.
C.
D.

An analysis of the curriculum of a given
school system.
A compilation of specific behavioral
management needs of a child with disabilities
A synthesis of the past educational
achievements of a child with disabilities
A compilation of life skills needed by a child
with disabilities in present or future settings.

Dr. Candace Baker

37
02/01/14
Q6 -- Answer
The

best answer is D.

 An

ecological inventory is designed to
determine those skills needed by a
particular individual in his or her current
and future environments.

Dr. Candace Baker

38
02/01/14
Q7. A student with behavior problems is being
taught to interact appropriately with peers on
the playground. The playground supervisor
provides the teacher with feedback on the
student’s behavior after every recess. The
student earns points for acceptable behavior.
When an infraction has occurred during recess,
the student loses points.
This contingent
withdrawal of reinforcers is referred to as . . .
A.
B.
C.
D.

negative reinforcement
discrimination learning
response cost
generalization training

Dr. Candace Baker

39
02/01/14
Q7 -- Answer
The

correct answer is C.

 Response

cost refers to the removal or
withdrawal of reinforcing stimuli upon the
performance of an undesirable behavior.

Dr. Candace Baker

40
02/01/14
Q8. Tomas, a student in a first-grade class,
is the only child standing quietly in a bus
line. His teacher asks Tomas to show the
other children how to stand quietly in line.
Many of the students imitate Tomas’
behavior. Which of the following behavior
management techniques did the teacher use?
A.
B.
C.
D.

Modeling
Shaping
Extinction
The Premack principle

Dr. Candace Baker

41
02/01/14
Q8 -- Answer
The

correct answer is A.

 Modeling

is a process by which an
observer learns a response or behavior by
watching and imitating the behavior of
another.

Dr. Candace Baker

42
02/01/14
Q9. As mandated in IDEA, the IEP team must consider
the need for transition services for each student before
she or he leaves school. If it is determined that
transition services are needed, which of the following
represents the most critical task for the team involved in
preparing the transition plan?
A.

B.

C.

D.

Adopting a transition plan previously drafted by officials of the
state department of education, the vocational rehabilitation
agency, and the local school district.
Getting information regarding post school educational services,
community living options, and employment, from adult agency
personnel.
Providing written information for the family and student to
review and suggesting a placement for postsecondary special
services.
Conducting meetings at which family and student goals are
determined, along with personnel responsibilities, information
to be collected, a time frame for providing information, and the
resources needed to meet each goal.

Dr. Candace Baker

43
02/01/14
Q9 -- Answer
The

best answer is D.

 Transition

to post school life will be most
effective when the goals of the student and
his or her family are considered, when all
responsible agencies work together to
develop a plan for the student, and when
specific tasks toward the goals are
scheduled, monitored, and completed.

Dr. Candace Baker

44
02/01/14
Q10. For a special education teacher, which
of the following is the best example of
collaborative goal setting?
A.

B.
C.
D.

Developing IEP goals with the regular
classroom teacher and then presenting the
completed goals to the students’ parents
Reaching a consensus on goals by consulting
with parents and the multidisciplinary team
Allowing parents to choose from goals
designed by the multidisciplinary team
Encouraging students to select goals for IEP
inclusion from teacher-approved lists.

Dr. Candace Baker

45
02/01/14
Q10 -- Answer
The

best answer is B,

 Since

collaborative goal setting requires the
sharing of ideas among all those involved
with the student – educators, parents, related
services providers, etc. It is not appropriate
in collaborative goal setting for one person or
group to present previously prepared goals to
the student or his or her family. There must
be collaboration among all concerned to
develop appropriate goals for the student.

Dr. Candace Baker

46
02/01/14
Special Education: KnowledgeBased Core Principles

Dr. Candace Baker

47
02/01/14
Q1. Which of the following techniques is
likely to be most successful in helping
learners with mental retardation to retain
previously acquired skills?
A.
B.
C.
D.

Scheduling frequent peer tutoring
sessions
Acknowledging appropriate behavior
regularly
Providing periodic review of lessons
Allowing longer independent practice
periods.

Dr. Candace Baker

48
02/01/14
Q1 -- Answer
The

best answer is C.

 Once

a skill has been mastered, the teacher can
best assure its retention by providing additional
meaningful learning situations in which the
student can recall and use the skill.

Dr. Candace Baker

49
02/01/14
Q2. Which of the following is most clearly
an example of a student using inappropriate
syntax?
A.

Saying, “I see football game”

B.

Saying, “Wa wa” as a substitute for water

C.

Saying, “Me sister shoes new happy”

D.

Saying, “He drinked his milk”

Dr. Candace Baker

50
02/01/14
Q2 -- Answer
best answer is C;
 Since using appropriate syntax involves
following rules for the correct arrangement
of word sequences.
 The

Dr. Candace Baker

51
02/01/14
Q3. Which of the following
conditions is frequently
attributable to Trisomy 21?
A.

Down syndrome

B.

Phenylketonuria (PKU)

C.

Klinefelter syndrome

D.

Turner syndrome

Dr. Candace Baker

52
02/01/14
Q3 -- Answer
correct answer is A.
 There are several possible causes of
Down syndrome, with the most common
being Trisomy 21, the failure of one pair
of parental chromosomes to separate at
conception.
 The

Dr. Candace Baker

53
02/01/14
Q4. According to IDEA, when parents
and schools disagree over education issues
for a child with a disability, either party
can request a . . .
A.

Mediator

B.

Due process hearing

C.

New teacher for the student

D.

Court date

Dr. Candace Baker

54
02/01/14
Q4 -- Answer
best answer is B.
According to IDEA, either a parent or
The

the public educational agency may
initiate a hearing on the identification,
evaluation, or educational placement
of a student with a disability.

Dr. Candace Baker

55
02/01/14
Q5. Which of the following placement
practices is required by federal law?
A.

Full inclusion

B.

Mainstreaming

C.

Least restrictive environment

D.

Resource room

Dr. Candace Baker

56
02/01/14
Q5 -- Answer
correct answer is C.
IDEA states that each student must be
The

placed
in
the
least
restrictive
environment in which the unique needs
of that student can be met. Choice A, B,
or D may be the least restrictive
environment for a given student, but that
must be determined on an individual
basis.

Dr. Candace Baker

57
02/01/14
Q6. According to IDEA, which of the
following is a mandated responsibility of the
school in servicing a student who has been
identified as having a disability?
A.
B.

C.
D.

Guaranteeing that the student is placed in a
regular classroom in the student’s home school
Paying for an independent educational
evaluation of the student upon request by the
parent
Notifying the parent following the placement of
the student in a special education program
Guaranteeing appropriate access to and
confidentiality of the student’s records.

Dr. Candace Baker

58
02/01/14
Q6 -- Answer
The

correct answer is D,

 As

both confidentiality of and access to
students’ records are addressed in IDEA.

Dr. Candace Baker

59
02/01/14
Q7. The most useful information for
developing the long-range goals and shortterm objectives on an IEP is likely to come
from using the results of which of the
following types of assessment?
A.
B.
C.
D.

Norm-referenced tests and adaptive behavior
scales
Criterion-referenced tests and behavioral
checklists
Norm-referenced tests and language samples
Developmental scales and group achievement
tests

Dr. Candace Baker

60
02/01/14
Q7 -- Answer
The

best answer is B.

 Criterion-referenced

tests compare a student’s
performance to a specified level of mastery or
achievement. Behavioral checklists may be
used to gather various types of information
about the student’s current performance in a
given area. Both assess specific information
about a particular student, and therefore can be
useful in developing IEP goals and
objectives.

Dr. Candace Baker

61
02/01/14
Q8. Which of the following best describes
the group of special education students for
whom transition plans must be developed
as part of the IEP process?
A.
B.
C.
D.

Students of all ages
Students between the ages of 18 and 21
Students aged 14 to 16 through school
completion
Students who will graduate from high
school and be placed in a sheltered
workshop

Dr. Candace Baker

62
02/01/14
Q8 -- Answer
correct answer is C, as stated in
IDEA.

The

Dr. Candace Baker

63
02/01/14
Q9. An IEP must include which of the
following components?
A.

The current level of student performance

B.

A record of past student performance

C.

A description of the student’s intellectual
functioning

D.

Suggestions for parental involvement

Dr. Candace Baker

64
02/01/14
Q9 -- Answer
correct answer is A, as stated in
IDEA.

The

Dr. Candace Baker

65
02/01/14
Special Education: Teaching
Students with Mental Retardation

Dr. Candace Baker

66
02/01/14
1. After scoring 100% on an oral multiplication
test and repeating this performance on a similar
paper-and-pencil test, a 12-year-old student with
mental retardation was unable to determine the
total cost of purchasing six comic books when the
price was $3 for one comic book. This student is
most clearly deficient in which of the following
areas?
A. Memory skills

B. Computational skills

C. Conceptual skills

C. Generalization skills

Dr. Candace Baker

67
02/01/14
Q1 -- Answer
best answer is D.
Poor generalization skills are exhibited
when an individual cannot use a learned
skill under conditions different from the
learning situation.
The

Dr. Candace Baker

68
02/01/14
Q2. Which of the following statements best
explains the increase in the number of children
identified as having mild mental retardation at
the time this population reaches school age?
A.
B.

C.
D.

Children with mental retardation tend to exhibit a
cognitive slump at about age 6.
Mental retardation is easier to detect at this age
because emphasis is placed on academic
achievement.
A label of mild mental retardation cannot legally be
given to children before they reach school age.
School personnel are generally more eager than
parents to find the reasons for children’s learning
difficulties.

Dr. Candace Baker

69
02/01/14
Q2 -- Answer
The

best answer is B.

 Young

children with mild mental retardation
often have age-appropriate adaptive behavior
and social skills, and therefore perform
adequately in the nonacademic situations
encountered before school begins. They begin
to exhibit problems, however, when the
academic demands of the classroom are placed
on them.

Dr. Candace Baker

70
02/01/14
Q3. According to the definition used by the
American Association on Mental Retardation
(AAMR), which of the following is one of the
conditions required before an individual can be
said to have mental retardation?
A.
B.
C.
D.

An IQ score below 50
A known cause of the individual’s condition
Deficits in adaptive behavior
Deficits in social skills

Dr. Candace Baker

71
02/01/14
Q3 -- Answer
the choices given, C is the only
condition specified in the AAMR
definition of mental retardation.

Of

Dr. Candace Baker

72
02/01/14
Q4.
The most recent conceptual thinking
related to program delivery options for individuals
with mental retardation gives the greatest amount
of emphasis to the…
A.
B.
C.
D.

development of skills that will promote
independence in an institutional setting
development of skills that will help with
integration into the community
mastery of academic skills
mastery of computer technology

Dr. Candace Baker

73
02/01/14
Q4 -- Answer

best answer is B.
Although the process of successful
integration into the community is
extremely complex both conceptually and
practically, it has been noted by many
educators, researchers, and advocacy
groups in recent years that it is essential
for appropriate programming to occur in
school to assure such success for
individuals with mental retardation.
The

Dr. Candace Baker

74
02/01/14
Q5. Using a functional approach is important
when designing instruction for students with
moderate and severe degrees of mental
retardation because this approach…
A.
B.

C.
D.

follows a sequence according to normal child
developmental milestones
breaks down the skills to be taught into a series of
discrete behaviors and then sequences those
behaviors into the progression in which they occur
when performing the skill
concentrates on behaviors that are critical to
performing a necessary task
teaches those skills that occur naturally within a
school environment.

Dr. Candace Baker

75
02/01/14
Q5 -- Answer
correct answer is C, since the
functional approach emphasizes
teaching skills that the students
need now or will need in the future.

The

Dr. Candace Baker

76
02/01/14
Q6. Which of the following procedures is
likely to be most effective in teaching basic
arithmetic concepts to students with mental
retardation?
A. Using manipulatives and pictures for
counting and grouping
B. Completing paper-and-pencil drilland-practice worksheets
C. Using calculators for problem-solving
D. Using flash cards for learning basic
addition and subtraction facts.
Dr. Candace Baker

77
02/01/14
Q6 -- Answer
 The

best answer is A, since counting and
grouping are two of the basic elements
required in learning arithmetic concepts. All
children who are learning basic arithmetic
concepts are assisted in this effort if
manipulatives, pictures, or other concrete
representations of the concepts are
presented.

Dr. Candace Baker

78
02/01/14
Professional Assessments
for Beginning Teachers
Special Education

79
(0381) Text Questions & Answers

02/01/14
Q1. Which of the following is the focus of the
noncategorical or cross-categorical approach to
special education?
A.
B.
C.
D.
E.

The use of self-contained special education
classes for students with disabilities
The differentiation of students’ needs based on
their specific disabilities
The education of students with disabilities based
on their similar characteristics and needs
The use of differential diagnosis as a technique
for distinguishing among disabilities
The application of task-analysis techniques in
the instruction of students with severe
disabilities.

Dr. Candace Baker

80
02/01/14
Q1 -- Answer
The

best answer is C.

 Schools

or districts that use a noncategorical or
cross-categorical approach to special education
generally base that decision on a belief that the
etiology of students’ disabilities is not
important. Rather, the students’ instructional
needs should be assessed, and students with
similar needs should receive similar treatment.

Dr. Candace Baker

81
02/01/14
Q2. Which of the following is a factor that impedes
researchers’ attempts to determine with certainty the
prevalence of learning disabilities in the United States?
A.
B.

C.

D.

E.

The absence of any definition of learning disabilities
in federal guidelines for special education
A general reluctance on the part of educators and
diagnosticians to classify students as having learning
disabilities
The overlap in identification criteria between the
classifications of severe learning disabilities and
severe emotional disturbance
The tendency of school districts to place students
identified as having learning disabilities in regular
rather than special education classes
The lack of a precisely defined cut-off point at which
a learning problem requiring remediation becomes a
disability requiring special education.
82

Dr. Candace Baker

02/01/14
Q2 -- Answer
 The

best answer is E.

 Definitions

of learning disabilities are often imprecise
in their guidance regarding the point at which a specific
learning problem should be classified as a disability, a
factor that affects identification of the condition.
Choice A is incorrect because the Individuals with
Disabilities Education Act defines specific learning
disabilities in general terms. B is incorrect; the
numbers indicate just the opposite. C is incorrect
because identification criteria for these two areas do not
present an appreciable problem of overlap. D is
irrelevant; placement is not the issue of the question.
Dr. Candace Baker

83
02/01/14
Q3. The total number of children with
disabilities who are served in special education
programs has grown at a phenomenal rate since
1975. Which of the following is the most likely
reason for the increase?
A.

B.
C.
D.
E.

The passage of PL 94-142, which required that the
public schools educate all children with disabilities
from age three through twenty-one
Improved assessment techniques that made it easier
to locate and identify children with disabilities
An increase in the number of special education
teachers trained to serve students with disabilities
A general decrease in school enrollment resulting
in more space for special education students
An increase in federal aid for education

Dr. Candace Baker

84
02/01/14
Q3 -- Answer
The

best answer is A.

 The

Education for All Handicapped Children
Act (PL 94-142), passed in 1975, was the
first law to mandate a free and appropriate
public education for all children and youth
with disabilities. Prior to that time, schools
could exclude children they felt were not
educable.

Dr. Candace Baker

85
02/01/14
Q4. Kevin is a second grader whose gradeequivalent score on a reading test is 5.0. Which
of the following is an accurate interpretation of
Kevin’s score?
A.
B.
C.
D.
E.

Kevin has the reading skills of the average fifth
grader.
Kevin can read fifth-grade materials.
Kevin correctly answered as many questions on the
test as did the average beginning fifth grader.
Kevin has progressed three years beyond the level at
which he is expected to be reading.
Kevin has mastered the skills of the reading
curriculum up to the fifth-grade level.
Dr. Candace Baker

86
02/01/14
Q4 -- Answer
The

best answer is C.

A

grade-equivalent score for an individual
represents the score on the test that is the
same as the median, or average, score for
all students at that grade level in the
norming group.
Therefore, if Kevin
obtained a grade-equivalent score of 5.0 on
a reading test, he correctly answered the
same number of questions on the test as did
the average beginning fifth grader.

Dr. Candace Baker

87
02/01/14
Q5. Of the following methods of assessment,
which is likely to provide the LEAST useful
information for instructional planning?
A.

Intelligence tests

B.

Behavioral checklists

C.

Criterion-referenced achievement tests

D.

Systematic observations

E.

Informal skill inventories

Dr. Candace Baker

88
02/01/14
Q5 -- Answer
The

best answer is A.

 It

has been widely acknowledged in the
field of education that, although intelligence
tests may serve several appropriate
purposes, they provide teachers with limited
useful information. Results of intelligence
tests are unlikely to help teachers decide
what to teach or how to teach it.

Dr. Candace Baker

89
02/01/14
Questions 6 – 8 are based on the
following information.
 Michelle,

a 19-year-old whose parents are
deceased, has profound mental retardation
and lives in a group home. She is bused to a
special class by the local school district.
Michelle has had frequent, extended absences
from school because of her frail physical
condition. She is nonambulatory and
nonverbal; her vision and hearing are intact.
She can communicate a few basic needs by
means of a pictorial language board.

Dr. Candace Baker

90
02/01/14
Q6. Which of the following is the
appropriate
focus
of
Michelle’s
individualized education program (IEP)?
A.

Functional academics

B.

Basic academic skills

C.

Prevocational skills

D.

Self-care skills

E.

Oral language

Dr. Candace Baker

91
02/01/14
Q6 -- Answer
The

best answer is D.

 Because

of Michelle’s profound level of
mental retardation and the severe nature of
her secondary disabilities, choices A, B, C,
and E are not likely to be realistic or useful
areas on which to focus. Self-care skills,
however, would help Michelle function better
both at school and at home.

Dr. Candace Baker

92
02/01/14
Q7. Michelle’s IEP would be expected
to include goals and objectives in all of
the following EXCEPT . . .
A.

Recreation

B.

Articulation

C.

Feeding

D.

Toileting

E.

Communication

Dr. Candace Baker

93
02/01/14
Q7 -- Answer
The

best answer is B.

 Michelle,

at age nineteen, is still
nonverbal. Therefore, articulation is an
inappropriate area for which to include
goals and objectives on her IEP.

Dr. Candace Baker

94
02/01/14
Q8. Since Michelle’s parents are deceased, who
is required to review and approve her
educational program?
A.
B.
C.
D.
E.

A representative of the state agency for children’s
services
A court-designated staff member of the group
home in which she resides
A blood relative willing to assume this
responsibility
A surrogate parent appointed by the local
education agency
A member of the local school board where
Michelle attends school

Dr. Candace Baker

95
02/01/14
Q8 -- Answer
best answer is D, as stated
in PL 94-142 and subsequent
legislation.

The

Dr. Candace Baker

96
02/01/14
Q9. Which of the following is most important for
teachers of students with disabilities to do during
scheduled conferences with parents?
A.
B.
C.
D.
E.

Use professional terminology when referring to the
student’s disability and instructional needs
Express agreement with the parents’ opinions about
their child’s strengths and weaknesses
Use the time in a leisurely, unstructured manner so
that issues and concerns will arise spontaneously
Determine whether the parents have hidden feelings
of frustration regarding their child’s disability
Present work samples and other evidences of the
student’s skills and discuss ideas for future activities.

Dr. Candace Baker

97
02/01/14
Q9 -- Answer
 The

best answer is E.

 Student

work samples and other evidence are useful
for giving parents a concrete representation of their
child’s progress, and teachers should include parents
in a discussion of educational planning. A would be
inappropriate. B is not an especially helpful policy in
itself and would be dishonest if agreement did not
exist. C is incorrect; a conference should be wellprepared so the time can be used productively. D
would be inappropriate.

Dr. Candace Baker

98
02/01/14
Q10.

A first-grade teacher has established a
system in which students earn points by complying
with classroom rules and completing work. The
teacher also takes points away fr0m students who
do not follow rules and do not complete work.
When taking away points, the teacher is using
which of the following techniques?
A.
B.
C.
D.
E.

Negative reinforcement
Overcorrection
Response cost
Timeout
Differential reinforcement

Dr. Candace Baker

99
02/01/14
Q10 -- Answer
The

best answer is C.

 Response

cost is a behavior-management
strategy in which a positive reinforcing
stimulus (in this case, earned points) is
removed following the performance of an
undesirable behavior.

Dr. Candace Baker

100
02/01/14
Educable Mentally
Handicapped

101
(0381) Text Questions & Answers

02/01/14
Q1. In relation to cognitive development, most
five-year-old children are able to:
A.
B.
C.
D.

Use numerical concepts to solve problems.
Identify abstract similarities and
differences.
Draw conclusions based on their own
experience.
Generalize rules from one situation to
another.

Dr. Candace Baker

102
02/01/14
Q1 -- Answer
 The


best answer is C.

In terms of their cognitive development, five-year-old children
typically exhibit the ability to draw conclusions based on their own
experience. The fact that their thinking is often characterized by a
number of faulty assumptions and conclusions is more a result of
their extremely limited experience than an inability to draw what are
to them reasonable conclusions. On the other hand, most five-yearolds are unable to use numerical concepts to solve problems (choice
A), identify abstract similarities and differences (choice B), or
understand how rules can be generalized from one situation to
another (choice D).
These cognitive abilities are generally
developed by children during middle and later childhood.

Dr. Candace Baker

103
02/01/14
Q2.

Piaget’s Theory is based on the premise that
children’s cognitive development depends primarily
upon:

A.
B.
C.
D.

The appropriate use of positive and negative
reinforcers by parents and other caretakers.
Their unconscious drives and motives.
Their attainment of a biologically based,
sequential series of developmental stages.
The availability of appropriate role models.

Dr. Candace Baker

104
02/01/14
Q2 -- Answer


The best answer is C.



This question requires knowledge of a major theory of human cognitive
development. According to Piagetian theory, intellectual development occurs
in a universal sequence of stages, each stage building on cognitive skills
attained during previous stages and involving progressively more complex
types of thought. The universality of the developmental stages observed by
Piaget in children worldwide caused him to hypothesize further that these
stages reflect an immutable and biologically based element common to all
human populations (choice C). The alternative choices listed all characterize
other well-known theories of development. The use of positive and negative
reinforcers (choice A) as primary external factors in cognitive development
characterizes behaviorist theories of development rather than Piaget’s theory.
A child’s unconscious drives and motives (choice B) are the underlying
factors in a variety of psychoanalytic approaches that focus on the
subconscious as a primary determinant of human development. Finally, the
availability of appropriate role models (choice D) is viewed as an important
causal factor in social learning theory but not in Piagetian theory.

Dr. Candace Baker

105
02/01/14
Q3. Which of the following cognitive disabilities
is caused by the presence of an extra
chromosome?

A.
B.
C.
D.

Tay-Sachs disease
Fragile X
Down syndrome
Phenylketonuria (PKU)

Dr. Candace Baker

106
02/01/14
Q3 -- Answer
 The


best answer is C.

Down syndrome is a cognitive disability caused by a genetic
abnormality involving the presence of an extra twenty-first
chromosome (choice C). Although the other conditions listed in the
question are also of genetic origin, none is caused by the presence of
an extra chromosome. Tay-Sachs disease (choice A) is the result of
a recessive gene. It causes rapid mental and physical deterioration
after birth and typically leads to death during early childhood.
Fragile X (choice B) involves an abnormality of the X chromosome
that results in mental impairment, particularly in males.
Phenylketonuria (PKU) (choice D) is caused by a recessive gene that
affects phenylalnin metabolism. If left untreated, PKU can also
cause a severe cognitive disability.
107
Dr. Candace Baker
02/01/14
Q4.

Mr. Bronson’s sixth grade class includes
students from several ethnic minority groups. To
plan an effective classroom management strategy,
it is important for Mr. Bronson to recognize that
these students:
A.
B.
C.
D.

Generally wish to assimilate into the dominant
culture.
Are likely to be openly hostile to a white male
teacher.
Will see one another as allies in overcoming
discrimination.
May form ethnic peer groups that become isolated
from another.
Dr. Candace Baker

108
02/01/14
Q4 -- Answer
The

best answer is D.

 In

typical social development, preadolescent children are
likely to form social groups based on perceived similarity
(choice D). A teacher wishing to promote ethnic diversity
cannot assume that members of various ethnic groups wish
to assimilate into the dominant culture (choice A).
Students will respond to the teacher on the basis of their
previous school and family experiences and will not
necessarily be hostile to a whit male teacher (choice B).
Because children of this age group tend to form social
groups based on obvious similarities, they will not
necessarily see one another as allies (choice C) unless they
are provided with instruction and structured experiences
that help them understand discrimination.
Dr. Candace Baker

109
02/01/14

Mais conteúdo relacionado

Mais procurados

Cognitive development on high school learners
Cognitive development on high school learnersCognitive development on high school learners
Cognitive development on high school learnerselockin24
 
Basic concept of learning disability psentation
Basic concept of learning disability psentationBasic concept of learning disability psentation
Basic concept of learning disability psentationDivina Bumacas
 
Learners with Exceptionalities
Learners with ExceptionalitiesLearners with Exceptionalities
Learners with ExceptionalitiesMirea Mizushima
 
Learners with Additional Needs.pptx
Learners with Additional Needs.pptxLearners with Additional Needs.pptx
Learners with Additional Needs.pptxRhodithMontealegre
 
SPED Classroom Management
SPED Classroom ManagementSPED Classroom Management
SPED Classroom ManagementSamantha Napil
 
Sped Presentation2
Sped Presentation2Sped Presentation2
Sped Presentation2johanndlee
 
Modifications and accommodations
Modifications and accommodationsModifications and accommodations
Modifications and accommodationsrgiaquinto
 
History of special education
History of special educationHistory of special education
History of special educationMadiha Rahman
 
Inclusive education system in Sped Philippine Setting
Inclusive education system in Sped Philippine SettingInclusive education system in Sped Philippine Setting
Inclusive education system in Sped Philippine SettingAlletMicaSTAANA
 
History of special education
History of special educationHistory of special education
History of special educationLaurenMarie887
 
Assessment of Students with Special Needs
Assessment of Students with Special NeedsAssessment of Students with Special Needs
Assessment of Students with Special Needsdani803
 
Educational Placement in Special Education
Educational Placement in Special EducationEducational Placement in Special Education
Educational Placement in Special EducationJewel Jem
 
Physical and Health Impairment
Physical and Health ImpairmentPhysical and Health Impairment
Physical and Health ImpairmentRita May Tagalog
 
Assessment of Learning - Multiple Choice Test
Assessment of Learning - Multiple Choice TestAssessment of Learning - Multiple Choice Test
Assessment of Learning - Multiple Choice TestXiTian Miran
 
Completion type of Test
Completion type of TestCompletion type of Test
Completion type of TestFatima Lara
 
Special education in the united states
Special education in the united statesSpecial education in the united states
Special education in the united statesMhia Maravilla
 
Diana v. state board of education 1970
Diana v. state board of education 1970Diana v. state board of education 1970
Diana v. state board of education 1970Carlos Sandoval
 
Special education timeline (1)
Special education timeline (1)Special education timeline (1)
Special education timeline (1)KimberlyHinkston
 

Mais procurados (20)

Cognitive development on high school learners
Cognitive development on high school learnersCognitive development on high school learners
Cognitive development on high school learners
 
Curriculum
CurriculumCurriculum
Curriculum
 
Basic concept of learning disability psentation
Basic concept of learning disability psentationBasic concept of learning disability psentation
Basic concept of learning disability psentation
 
Learners with Exceptionalities
Learners with ExceptionalitiesLearners with Exceptionalities
Learners with Exceptionalities
 
Learners with Additional Needs.pptx
Learners with Additional Needs.pptxLearners with Additional Needs.pptx
Learners with Additional Needs.pptx
 
SPED Classroom Management
SPED Classroom ManagementSPED Classroom Management
SPED Classroom Management
 
Sped Presentation2
Sped Presentation2Sped Presentation2
Sped Presentation2
 
Modifications and accommodations
Modifications and accommodationsModifications and accommodations
Modifications and accommodations
 
History of special education
History of special educationHistory of special education
History of special education
 
Inclusive education system in Sped Philippine Setting
Inclusive education system in Sped Philippine SettingInclusive education system in Sped Philippine Setting
Inclusive education system in Sped Philippine Setting
 
History of special education
History of special educationHistory of special education
History of special education
 
Assessment of Students with Special Needs
Assessment of Students with Special NeedsAssessment of Students with Special Needs
Assessment of Students with Special Needs
 
Educational Placement in Special Education
Educational Placement in Special EducationEducational Placement in Special Education
Educational Placement in Special Education
 
Physical and Health Impairment
Physical and Health ImpairmentPhysical and Health Impairment
Physical and Health Impairment
 
Assessment of Learning - Multiple Choice Test
Assessment of Learning - Multiple Choice TestAssessment of Learning - Multiple Choice Test
Assessment of Learning - Multiple Choice Test
 
Completion type of Test
Completion type of TestCompletion type of Test
Completion type of Test
 
Special education in the united states
Special education in the united statesSpecial education in the united states
Special education in the united states
 
Special education and inclusion
Special education and inclusionSpecial education and inclusion
Special education and inclusion
 
Diana v. state board of education 1970
Diana v. state board of education 1970Diana v. state board of education 1970
Diana v. state board of education 1970
 
Special education timeline (1)
Special education timeline (1)Special education timeline (1)
Special education timeline (1)
 

Semelhante a Lbs1 practice test

I- Multiple Choice Questions (10 points)Please underline the
I- Multiple Choice Questions (10 points)Please underline theI- Multiple Choice Questions (10 points)Please underline the
I- Multiple Choice Questions (10 points)Please underline theLizbethQuinonez813
 
What are the disadvantages of standardizes testing
What are the disadvantages of standardizes testingWhat are the disadvantages of standardizes testing
What are the disadvantages of standardizes testingumarfarooq915
 
Group 6 Chapter Presentation Powerpoint
Group 6 Chapter Presentation PowerpointGroup 6 Chapter Presentation Powerpoint
Group 6 Chapter Presentation Powerpointkim_coop
 
Assessing students with learning disabilities Yeth
Assessing students with learning disabilities YethAssessing students with learning disabilities Yeth
Assessing students with learning disabilities YethGu Luchavez
 
30 Methods To Improve Learning Capability In Slow Learners
30 Methods To Improve Learning Capability In Slow Learners30 Methods To Improve Learning Capability In Slow Learners
30 Methods To Improve Learning Capability In Slow LearnersLeslie Schulte
 
PSY 475 Week 3 Individual Assignment Attitude Survey1 2015 version
PSY 475 Week 3 Individual Assignment Attitude Survey1 2015 versionPSY 475 Week 3 Individual Assignment Attitude Survey1 2015 version
PSY 475 Week 3 Individual Assignment Attitude Survey1 2015 versionbailieshiramizu
 
PSY 475 Week 3 Individual Assignment Attitude Survey1 2015 version
PSY 475 Week 3 Individual Assignment Attitude Survey1 2015 versionPSY 475 Week 3 Individual Assignment Attitude Survey1 2015 version
PSY 475 Week 3 Individual Assignment Attitude Survey1 2015 versionbailieshiramizu
 
PSY 475 Week 3 Individual Assignment Attitude Survey1 2015 version
PSY 475 Week 3 Individual Assignment Attitude Survey1 2015 versionPSY 475 Week 3 Individual Assignment Attitude Survey1 2015 version
PSY 475 Week 3 Individual Assignment Attitude Survey1 2015 versionbailieshiramizu
 
PSY 475 Week 3 Individual Assignment Attitude Survey1 2015 version
PSY 475 Week 3 Individual Assignment Attitude Survey1 2015 versionPSY 475 Week 3 Individual Assignment Attitude Survey1 2015 version
PSY 475 Week 3 Individual Assignment Attitude Survey1 2015 versionwinnycasia
 
PSY 475 Week 3 Individual Assignment Attitude Survey1 2015 version
PSY 475 Week 3 Individual Assignment Attitude Survey1 2015 versionPSY 475 Week 3 Individual Assignment Attitude Survey1 2015 version
PSY 475 Week 3 Individual Assignment Attitude Survey1 2015 versionbailieshiramizu
 
PSY 475 Week 3 Individual Assignment Attitude Survey1 2015 version
PSY 475 Week 3 Individual Assignment Attitude Survey1 2015 versionPSY 475 Week 3 Individual Assignment Attitude Survey1 2015 version
PSY 475 Week 3 Individual Assignment Attitude Survey1 2015 versionwinnycasia
 
PSY 475 Week 3 Individual Assignment Attitude Survey1 2015 version
PSY 475 Week 3 Individual Assignment Attitude Survey1 2015 versionPSY 475 Week 3 Individual Assignment Attitude Survey1 2015 version
PSY 475 Week 3 Individual Assignment Attitude Survey1 2015 versionbailieshiramizu
 
PSY 475 Week 3 Individual Assignment Attitude Survey1 2015 version
PSY 475 Week 3 Individual Assignment Attitude Survey1 2015 versionPSY 475 Week 3 Individual Assignment Attitude Survey1 2015 version
PSY 475 Week 3 Individual Assignment Attitude Survey1 2015 versionwinnycasia
 
PSY 475 Week 3 Individual Assignment Attitude Survey1 2015 version
PSY 475 Week 3 Individual Assignment Attitude Survey1 2015 versionPSY 475 Week 3 Individual Assignment Attitude Survey1 2015 version
PSY 475 Week 3 Individual Assignment Attitude Survey1 2015 versionwinnycasia
 
PSY 475 Week 3 Individual Assignment Attitude Survey1 2015 version
PSY 475 Week 3 Individual Assignment Attitude Survey1 2015 versionPSY 475 Week 3 Individual Assignment Attitude Survey1 2015 version
PSY 475 Week 3 Individual Assignment Attitude Survey1 2015 versionwinnycasia
 
Least Restrictive Environment Presentation
Least Restrictive Environment PresentationLeast Restrictive Environment Presentation
Least Restrictive Environment Presentation6stahls
 
EDUC 102 – FOUNDATIONS OF SPECIAL AND INCLUSIVE - STUDENTS' CASES.pptx
EDUC 102 – FOUNDATIONS OF SPECIAL AND INCLUSIVE - STUDENTS' CASES.pptxEDUC 102 – FOUNDATIONS OF SPECIAL AND INCLUSIVE - STUDENTS' CASES.pptx
EDUC 102 – FOUNDATIONS OF SPECIAL AND INCLUSIVE - STUDENTS' CASES.pptxJustinPaulVallinan1
 
Backward child
Backward childBackward child
Backward childRubiRoy1
 

Semelhante a Lbs1 practice test (20)

I- Multiple Choice Questions (10 points)Please underline the
I- Multiple Choice Questions (10 points)Please underline theI- Multiple Choice Questions (10 points)Please underline the
I- Multiple Choice Questions (10 points)Please underline the
 
What are the disadvantages of standardizes testing
What are the disadvantages of standardizes testingWhat are the disadvantages of standardizes testing
What are the disadvantages of standardizes testing
 
Group 6 Chapter Presentation Powerpoint
Group 6 Chapter Presentation PowerpointGroup 6 Chapter Presentation Powerpoint
Group 6 Chapter Presentation Powerpoint
 
Dayana
DayanaDayana
Dayana
 
Assessing students with learning disabilities Yeth
Assessing students with learning disabilities YethAssessing students with learning disabilities Yeth
Assessing students with learning disabilities Yeth
 
30 Methods To Improve Learning Capability In Slow Learners
30 Methods To Improve Learning Capability In Slow Learners30 Methods To Improve Learning Capability In Slow Learners
30 Methods To Improve Learning Capability In Slow Learners
 
PSY 475 Week 3 Individual Assignment Attitude Survey1 2015 version
PSY 475 Week 3 Individual Assignment Attitude Survey1 2015 versionPSY 475 Week 3 Individual Assignment Attitude Survey1 2015 version
PSY 475 Week 3 Individual Assignment Attitude Survey1 2015 version
 
PSY 475 Week 3 Individual Assignment Attitude Survey1 2015 version
PSY 475 Week 3 Individual Assignment Attitude Survey1 2015 versionPSY 475 Week 3 Individual Assignment Attitude Survey1 2015 version
PSY 475 Week 3 Individual Assignment Attitude Survey1 2015 version
 
PSY 475 Week 3 Individual Assignment Attitude Survey1 2015 version
PSY 475 Week 3 Individual Assignment Attitude Survey1 2015 versionPSY 475 Week 3 Individual Assignment Attitude Survey1 2015 version
PSY 475 Week 3 Individual Assignment Attitude Survey1 2015 version
 
PSY 475 Week 3 Individual Assignment Attitude Survey1 2015 version
PSY 475 Week 3 Individual Assignment Attitude Survey1 2015 versionPSY 475 Week 3 Individual Assignment Attitude Survey1 2015 version
PSY 475 Week 3 Individual Assignment Attitude Survey1 2015 version
 
PSY 475 Week 3 Individual Assignment Attitude Survey1 2015 version
PSY 475 Week 3 Individual Assignment Attitude Survey1 2015 versionPSY 475 Week 3 Individual Assignment Attitude Survey1 2015 version
PSY 475 Week 3 Individual Assignment Attitude Survey1 2015 version
 
PSY 475 Week 3 Individual Assignment Attitude Survey1 2015 version
PSY 475 Week 3 Individual Assignment Attitude Survey1 2015 versionPSY 475 Week 3 Individual Assignment Attitude Survey1 2015 version
PSY 475 Week 3 Individual Assignment Attitude Survey1 2015 version
 
PSY 475 Week 3 Individual Assignment Attitude Survey1 2015 version
PSY 475 Week 3 Individual Assignment Attitude Survey1 2015 versionPSY 475 Week 3 Individual Assignment Attitude Survey1 2015 version
PSY 475 Week 3 Individual Assignment Attitude Survey1 2015 version
 
PSY 475 Week 3 Individual Assignment Attitude Survey1 2015 version
PSY 475 Week 3 Individual Assignment Attitude Survey1 2015 versionPSY 475 Week 3 Individual Assignment Attitude Survey1 2015 version
PSY 475 Week 3 Individual Assignment Attitude Survey1 2015 version
 
PSY 475 Week 3 Individual Assignment Attitude Survey1 2015 version
PSY 475 Week 3 Individual Assignment Attitude Survey1 2015 versionPSY 475 Week 3 Individual Assignment Attitude Survey1 2015 version
PSY 475 Week 3 Individual Assignment Attitude Survey1 2015 version
 
PSY 475 Week 3 Individual Assignment Attitude Survey1 2015 version
PSY 475 Week 3 Individual Assignment Attitude Survey1 2015 versionPSY 475 Week 3 Individual Assignment Attitude Survey1 2015 version
PSY 475 Week 3 Individual Assignment Attitude Survey1 2015 version
 
Least Restrictive Environment Presentation
Least Restrictive Environment PresentationLeast Restrictive Environment Presentation
Least Restrictive Environment Presentation
 
EDUC 102 – FOUNDATIONS OF SPECIAL AND INCLUSIVE - STUDENTS' CASES.pptx
EDUC 102 – FOUNDATIONS OF SPECIAL AND INCLUSIVE - STUDENTS' CASES.pptxEDUC 102 – FOUNDATIONS OF SPECIAL AND INCLUSIVE - STUDENTS' CASES.pptx
EDUC 102 – FOUNDATIONS OF SPECIAL AND INCLUSIVE - STUDENTS' CASES.pptx
 
Gifted Presentation
Gifted PresentationGifted Presentation
Gifted Presentation
 
Backward child
Backward childBackward child
Backward child
 

Mais de Jeffery Massey

Research proposal final pptx 1
Research proposal final pptx 1Research proposal final pptx 1
Research proposal final pptx 1Jeffery Massey
 
Educational trend ppt team review draft final 4 pdf
Educational trend ppt team review draft final 4 pdfEducational trend ppt team review draft final 4 pdf
Educational trend ppt team review draft final 4 pdfJeffery Massey
 
Press relase 2 course .project
Press relase 2 course .projectPress relase 2 course .project
Press relase 2 course .projectJeffery Massey
 
Newsletter 2.fina lpdf
Newsletter 2.fina lpdfNewsletter 2.fina lpdf
Newsletter 2.fina lpdfJeffery Massey
 
Newsletter course project 3
Newsletter course project 3Newsletter course project 3
Newsletter course project 3Jeffery Massey
 
Assessment plan blueprint 1 final for livetext
Assessment plan blueprint 1 final for livetextAssessment plan blueprint 1 final for livetext
Assessment plan blueprint 1 final for livetextJeffery Massey
 
Chicagoresearchroundtablepresentationfinalversion 110707151020-phpapp02
Chicagoresearchroundtablepresentationfinalversion 110707151020-phpapp02Chicagoresearchroundtablepresentationfinalversion 110707151020-phpapp02
Chicagoresearchroundtablepresentationfinalversion 110707151020-phpapp02Jeffery Massey
 
St. Sabina. Press. Release.031711
St. Sabina. Press. Release.031711St. Sabina. Press. Release.031711
St. Sabina. Press. Release.031711Jeffery Massey
 
Jeffery Massey Week 2 Learning Style Show 4 with anime
Jeffery Massey Week 2 Learning Style Show 4 with animeJeffery Massey Week 2 Learning Style Show 4 with anime
Jeffery Massey Week 2 Learning Style Show 4 with animeJeffery Massey
 
Educational Trend PPT Team Review Draft FINAL 4
Educational Trend PPT Team Review Draft FINAL 4Educational Trend PPT Team Review Draft FINAL 4
Educational Trend PPT Team Review Draft FINAL 4Jeffery Massey
 
Hpsp Scholarship Air Force
Hpsp Scholarship  Air  ForceHpsp Scholarship  Air  Force
Hpsp Scholarship Air ForceJeffery Massey
 
Icarus%20 Slide%20 Presentation1c[1]2
Icarus%20 Slide%20 Presentation1c[1]2Icarus%20 Slide%20 Presentation1c[1]2
Icarus%20 Slide%20 Presentation1c[1]2Jeffery Massey
 
Ron Ansd Jeff Compile 2 Chicago Christian Industrial League
Ron Ansd  Jeff  Compile 2  Chicago  Christian  Industrial  LeagueRon Ansd  Jeff  Compile 2  Chicago  Christian  Industrial  League
Ron Ansd Jeff Compile 2 Chicago Christian Industrial LeagueJeffery Massey
 
G T C N Exec Summ Program Description 1 R F P
G T C N  Exec  Summ  Program  Description 1  R F PG T C N  Exec  Summ  Program  Description 1  R F P
G T C N Exec Summ Program Description 1 R F PJeffery Massey
 
G T C N Exec Summ J M 1
G T C N  Exec  Summ  J M 1G T C N  Exec  Summ  J M 1
G T C N Exec Summ J M 1Jeffery Massey
 

Mais de Jeffery Massey (20)

Research proposal final pptx 1
Research proposal final pptx 1Research proposal final pptx 1
Research proposal final pptx 1
 
Icarus re save
Icarus re saveIcarus re save
Icarus re save
 
Educational trend ppt team review draft final 4 pdf
Educational trend ppt team review draft final 4 pdfEducational trend ppt team review draft final 4 pdf
Educational trend ppt team review draft final 4 pdf
 
Press relase 2 course .project
Press relase 2 course .projectPress relase 2 course .project
Press relase 2 course .project
 
Newsletter 2.fina lpdf
Newsletter 2.fina lpdfNewsletter 2.fina lpdf
Newsletter 2.fina lpdf
 
Newsletter course project 3
Newsletter course project 3Newsletter course project 3
Newsletter course project 3
 
Assessment plan blueprint 1 final for livetext
Assessment plan blueprint 1 final for livetextAssessment plan blueprint 1 final for livetext
Assessment plan blueprint 1 final for livetext
 
Jm watchlist 1 7.4.11
Jm watchlist 1 7.4.11Jm watchlist 1 7.4.11
Jm watchlist 1 7.4.11
 
Chicagoresearchroundtablepresentationfinalversion 110707151020-phpapp02
Chicagoresearchroundtablepresentationfinalversion 110707151020-phpapp02Chicagoresearchroundtablepresentationfinalversion 110707151020-phpapp02
Chicagoresearchroundtablepresentationfinalversion 110707151020-phpapp02
 
P D S G Flyer
P D  S G  FlyerP D  S G  Flyer
P D S G Flyer
 
St. Sabina. Press. Release.031711
St. Sabina. Press. Release.031711St. Sabina. Press. Release.031711
St. Sabina. Press. Release.031711
 
Ron Barnes Modiify
Ron Barnes ModiifyRon Barnes Modiify
Ron Barnes Modiify
 
Jeffery Massey Week 2 Learning Style Show 4 with anime
Jeffery Massey Week 2 Learning Style Show 4 with animeJeffery Massey Week 2 Learning Style Show 4 with anime
Jeffery Massey Week 2 Learning Style Show 4 with anime
 
Educational Trend PPT Team Review Draft FINAL 4
Educational Trend PPT Team Review Draft FINAL 4Educational Trend PPT Team Review Draft FINAL 4
Educational Trend PPT Team Review Draft FINAL 4
 
Hpsp Scholarship Air Force
Hpsp Scholarship  Air  ForceHpsp Scholarship  Air  Force
Hpsp Scholarship Air Force
 
Detective P P T 1
Detective  P P T 1Detective  P P T 1
Detective P P T 1
 
Icarus%20 Slide%20 Presentation1c[1]2
Icarus%20 Slide%20 Presentation1c[1]2Icarus%20 Slide%20 Presentation1c[1]2
Icarus%20 Slide%20 Presentation1c[1]2
 
Ron Ansd Jeff Compile 2 Chicago Christian Industrial League
Ron Ansd  Jeff  Compile 2  Chicago  Christian  Industrial  LeagueRon Ansd  Jeff  Compile 2  Chicago  Christian  Industrial  League
Ron Ansd Jeff Compile 2 Chicago Christian Industrial League
 
G T C N Exec Summ Program Description 1 R F P
G T C N  Exec  Summ  Program  Description 1  R F PG T C N  Exec  Summ  Program  Description 1  R F P
G T C N Exec Summ Program Description 1 R F P
 
G T C N Exec Summ J M 1
G T C N  Exec  Summ  J M 1G T C N  Exec  Summ  J M 1
G T C N Exec Summ J M 1
 

Último

Arihant handbook biology for class 11 .pdf
Arihant handbook biology for class 11 .pdfArihant handbook biology for class 11 .pdf
Arihant handbook biology for class 11 .pdfchloefrazer622
 
Activity 01 - Artificial Culture (1).pdf
Activity 01 - Artificial Culture (1).pdfActivity 01 - Artificial Culture (1).pdf
Activity 01 - Artificial Culture (1).pdfciinovamais
 
Hybridoma Technology ( Production , Purification , and Application )
Hybridoma Technology  ( Production , Purification , and Application  ) Hybridoma Technology  ( Production , Purification , and Application  )
Hybridoma Technology ( Production , Purification , and Application ) Sakshi Ghasle
 
Nutritional Needs Presentation - HLTH 104
Nutritional Needs Presentation - HLTH 104Nutritional Needs Presentation - HLTH 104
Nutritional Needs Presentation - HLTH 104misteraugie
 
Call Girls in Dwarka Mor Delhi Contact Us 9654467111
Call Girls in Dwarka Mor Delhi Contact Us 9654467111Call Girls in Dwarka Mor Delhi Contact Us 9654467111
Call Girls in Dwarka Mor Delhi Contact Us 9654467111Sapana Sha
 
Z Score,T Score, Percential Rank and Box Plot Graph
Z Score,T Score, Percential Rank and Box Plot GraphZ Score,T Score, Percential Rank and Box Plot Graph
Z Score,T Score, Percential Rank and Box Plot GraphThiyagu K
 
Advanced Views - Calendar View in Odoo 17
Advanced Views - Calendar View in Odoo 17Advanced Views - Calendar View in Odoo 17
Advanced Views - Calendar View in Odoo 17Celine George
 
A Critique of the Proposed National Education Policy Reform
A Critique of the Proposed National Education Policy ReformA Critique of the Proposed National Education Policy Reform
A Critique of the Proposed National Education Policy ReformChameera Dedduwage
 
microwave assisted reaction. General introduction
microwave assisted reaction. General introductionmicrowave assisted reaction. General introduction
microwave assisted reaction. General introductionMaksud Ahmed
 
CARE OF CHILD IN INCUBATOR..........pptx
CARE OF CHILD IN INCUBATOR..........pptxCARE OF CHILD IN INCUBATOR..........pptx
CARE OF CHILD IN INCUBATOR..........pptxGaneshChakor2
 
Q4-W6-Restating Informational Text Grade 3
Q4-W6-Restating Informational Text Grade 3Q4-W6-Restating Informational Text Grade 3
Q4-W6-Restating Informational Text Grade 3JemimahLaneBuaron
 
18-04-UA_REPORT_MEDIALITERAСY_INDEX-DM_23-1-final-eng.pdf
18-04-UA_REPORT_MEDIALITERAСY_INDEX-DM_23-1-final-eng.pdf18-04-UA_REPORT_MEDIALITERAСY_INDEX-DM_23-1-final-eng.pdf
18-04-UA_REPORT_MEDIALITERAСY_INDEX-DM_23-1-final-eng.pdfssuser54595a
 
URLs and Routing in the Odoo 17 Website App
URLs and Routing in the Odoo 17 Website AppURLs and Routing in the Odoo 17 Website App
URLs and Routing in the Odoo 17 Website AppCeline George
 
Grant Readiness 101 TechSoup and Remy Consulting
Grant Readiness 101 TechSoup and Remy ConsultingGrant Readiness 101 TechSoup and Remy Consulting
Grant Readiness 101 TechSoup and Remy ConsultingTechSoup
 
Kisan Call Centre - To harness potential of ICT in Agriculture by answer farm...
Kisan Call Centre - To harness potential of ICT in Agriculture by answer farm...Kisan Call Centre - To harness potential of ICT in Agriculture by answer farm...
Kisan Call Centre - To harness potential of ICT in Agriculture by answer farm...Krashi Coaching
 
SOCIAL AND HISTORICAL CONTEXT - LFTVD.pptx
SOCIAL AND HISTORICAL CONTEXT - LFTVD.pptxSOCIAL AND HISTORICAL CONTEXT - LFTVD.pptx
SOCIAL AND HISTORICAL CONTEXT - LFTVD.pptxiammrhaywood
 
Mastering the Unannounced Regulatory Inspection
Mastering the Unannounced Regulatory InspectionMastering the Unannounced Regulatory Inspection
Mastering the Unannounced Regulatory InspectionSafetyChain Software
 
mini mental status format.docx
mini    mental       status     format.docxmini    mental       status     format.docx
mini mental status format.docxPoojaSen20
 
Introduction to AI in Higher Education_draft.pptx
Introduction to AI in Higher Education_draft.pptxIntroduction to AI in Higher Education_draft.pptx
Introduction to AI in Higher Education_draft.pptxpboyjonauth
 

Último (20)

Staff of Color (SOC) Retention Efforts DDSD
Staff of Color (SOC) Retention Efforts DDSDStaff of Color (SOC) Retention Efforts DDSD
Staff of Color (SOC) Retention Efforts DDSD
 
Arihant handbook biology for class 11 .pdf
Arihant handbook biology for class 11 .pdfArihant handbook biology for class 11 .pdf
Arihant handbook biology for class 11 .pdf
 
Activity 01 - Artificial Culture (1).pdf
Activity 01 - Artificial Culture (1).pdfActivity 01 - Artificial Culture (1).pdf
Activity 01 - Artificial Culture (1).pdf
 
Hybridoma Technology ( Production , Purification , and Application )
Hybridoma Technology  ( Production , Purification , and Application  ) Hybridoma Technology  ( Production , Purification , and Application  )
Hybridoma Technology ( Production , Purification , and Application )
 
Nutritional Needs Presentation - HLTH 104
Nutritional Needs Presentation - HLTH 104Nutritional Needs Presentation - HLTH 104
Nutritional Needs Presentation - HLTH 104
 
Call Girls in Dwarka Mor Delhi Contact Us 9654467111
Call Girls in Dwarka Mor Delhi Contact Us 9654467111Call Girls in Dwarka Mor Delhi Contact Us 9654467111
Call Girls in Dwarka Mor Delhi Contact Us 9654467111
 
Z Score,T Score, Percential Rank and Box Plot Graph
Z Score,T Score, Percential Rank and Box Plot GraphZ Score,T Score, Percential Rank and Box Plot Graph
Z Score,T Score, Percential Rank and Box Plot Graph
 
Advanced Views - Calendar View in Odoo 17
Advanced Views - Calendar View in Odoo 17Advanced Views - Calendar View in Odoo 17
Advanced Views - Calendar View in Odoo 17
 
A Critique of the Proposed National Education Policy Reform
A Critique of the Proposed National Education Policy ReformA Critique of the Proposed National Education Policy Reform
A Critique of the Proposed National Education Policy Reform
 
microwave assisted reaction. General introduction
microwave assisted reaction. General introductionmicrowave assisted reaction. General introduction
microwave assisted reaction. General introduction
 
CARE OF CHILD IN INCUBATOR..........pptx
CARE OF CHILD IN INCUBATOR..........pptxCARE OF CHILD IN INCUBATOR..........pptx
CARE OF CHILD IN INCUBATOR..........pptx
 
Q4-W6-Restating Informational Text Grade 3
Q4-W6-Restating Informational Text Grade 3Q4-W6-Restating Informational Text Grade 3
Q4-W6-Restating Informational Text Grade 3
 
18-04-UA_REPORT_MEDIALITERAСY_INDEX-DM_23-1-final-eng.pdf
18-04-UA_REPORT_MEDIALITERAСY_INDEX-DM_23-1-final-eng.pdf18-04-UA_REPORT_MEDIALITERAСY_INDEX-DM_23-1-final-eng.pdf
18-04-UA_REPORT_MEDIALITERAСY_INDEX-DM_23-1-final-eng.pdf
 
URLs and Routing in the Odoo 17 Website App
URLs and Routing in the Odoo 17 Website AppURLs and Routing in the Odoo 17 Website App
URLs and Routing in the Odoo 17 Website App
 
Grant Readiness 101 TechSoup and Remy Consulting
Grant Readiness 101 TechSoup and Remy ConsultingGrant Readiness 101 TechSoup and Remy Consulting
Grant Readiness 101 TechSoup and Remy Consulting
 
Kisan Call Centre - To harness potential of ICT in Agriculture by answer farm...
Kisan Call Centre - To harness potential of ICT in Agriculture by answer farm...Kisan Call Centre - To harness potential of ICT in Agriculture by answer farm...
Kisan Call Centre - To harness potential of ICT in Agriculture by answer farm...
 
SOCIAL AND HISTORICAL CONTEXT - LFTVD.pptx
SOCIAL AND HISTORICAL CONTEXT - LFTVD.pptxSOCIAL AND HISTORICAL CONTEXT - LFTVD.pptx
SOCIAL AND HISTORICAL CONTEXT - LFTVD.pptx
 
Mastering the Unannounced Regulatory Inspection
Mastering the Unannounced Regulatory InspectionMastering the Unannounced Regulatory Inspection
Mastering the Unannounced Regulatory Inspection
 
mini mental status format.docx
mini    mental       status     format.docxmini    mental       status     format.docx
mini mental status format.docx
 
Introduction to AI in Higher Education_draft.pptx
Introduction to AI in Higher Education_draft.pptxIntroduction to AI in Higher Education_draft.pptx
Introduction to AI in Higher Education_draft.pptx
 

Lbs1 practice test

  • 1. LBS1 Practice Test for Saint Xavier University Candidates Dr. Candace Baker 1 02/01/14
  • 2. Special Education: Teaching Students with Learning Disabilities Dr. Candace Baker 2 02/01/14
  • 3. Q1. Students with learning disabilities are more likely than students without disabilities to exhibit which of the following? A. B. C. D. Clear Dominance of the left Brain for Learning Deficits in long-term memory retrieval Selective attention disorders Characteristics of giftedness in artistic expression Dr. Candace Baker 3 02/01/14
  • 4. Q1 -- Answer  The Correct Answer is C.  Research indicates that students with learning disabilities are likely to lag behind other children in their ability to identify and focus on the key aspects of a task while disregarding the less important aspects. It has not been proven that students with learning disabilities are more likely than other students to have either left-brain or right-brain dominance for learning, and students with learning disabilities have no particular difficulty with long-term memory retrieval (Choice B). There is no clear evidence to suggest that students with learning disabilities are more gifted than others in artistic expression (Choice D). Dr. Candace Baker 4 02/01/14
  • 5. Q2. Which of the following best represents current research findings on brain damage / dysfunction as it relates to learning disabilities? A. B. C. D. Direct, causal links have been found between prenatal exposure to toxic substances and certain well-defined learning disabilities. Brain damage/dysfunction has been ruled out altogether as a factor in the occurrence of learning disabilities. Brain damage/dysfunction caused by accident or abuse is clearly linked to learning disabilities, but such a link has not been established for damage that is prenatal. There is little direct evidence of a link between learning disabilities and brain damage/dysfunction but some evidence that is correlational. Dr. Candace Baker 5 02/01/14
  • 6. Q2 -- Answer Best Answer is D.  Specific causal linkages between pre- or postnatal development and/or experiences and learning disabilities, as indicated in choice A and choice C, have not yet been established with certainty. Because research is ongoing in this area, choice B is not correct.  The Dr. Candace Baker 6 02/01/14
  • 7. Q3. Which of the following single indicators is most commonly used to signify students’ eligibility for services for specific learning disabilities? A. B. C. D. Academic Achievement that is hampered by social and emotional deficits Academic achievement that is significantly below grade level. Academic aptitude that is significantly lower than the norm for the chronological age. Academic aptitude that is significantly higher than academic achievement. Dr. Candace Baker 7 02/01/14
  • 8. Q3 -- Answer Best Answer is D.  Although definitions of learning disabilities vary, most have an aptitude-achievement discrepancy component. Social and emotional deficits (choice A) are not primary to identifying learning disabilities. Choice B as a single indicator is incorrect; many factors other than the presence of a learning disability could account for low academic achievement. Chronological age related to aptitude (choice C) is not a primary indicator for learning disabilities.  The Dr. Candace Baker 8 02/01/14
  • 9. Q4. A Student with learning disabilities who reads at grade level is enrolled in a regular biology class. The special education teacher is trying to determine what strategies to use to help the student perform well in the class and pass the required tests. Which of the following Steps should the special education teacher take first? A. B. C. D. Determine how well the student organizes and retains material that is representative of the course. Obtain the required reading materials and highlight important concepts for the student. Secure a tape recorder for the student to use to tape the lectures and other teacher presentations. Use a readability index to determine the difficulty of the required reading materials. Dr. Candace Baker 9 02/01/14
  • 10. Q4 -- Answer  The correct answer is A.  Many students with learning disabilities have difficulty organizing materials for study and using strategies that aid their retention of information. As a first step, highlighting information (choice B) would not be useful; it would not provide information about the student, who might not have a deficit that requires this step. Choice C would not be useful as a first step, before the teacher learns the student’s specific study needs. Because the student reads at grade level it would be more advantageous to determine how the student approaches study tasks than to determine the difficulty of the material (Choice D). Dr. Candace Baker 10 02/01/14
  • 11. Q5. Ralph is a fourth grade, recently mainstreamed student who has a learning disability that causes him to decode written materials in a slow and labored manner. His comprehension is excellent. His teacher has organized the classroom in five reading / discussion groups to which students have been assigned based on their reading performance. Ralph is placed in the highest group. Which of the following best justifies the teacher’s decision? A. B. C. D. Ralph will have his strengths reinforced and will likely receive spontaneous peer tutoring. Ralph will be forced to decode faster in order to be competitive with the other students. The social benefits of being in the higher group will outweigh any instructional disadvantage Ralph experiences. Ralph can keep pace by listening to the discussion and thereby avoid some of the decoding tasks. Dr. Candace Baker 11 02/01/14
  • 12. Q5 -- Answer  The Best Answer is A.  Because Ralph has no trouble understanding what he reads, he is more likely to improve his skills in the highest group than in one in which the students have difficulty with both decoding and comprehension. The teacher can employ strategies that will aid him in decoding the materials. Trying to decode faster without any type of adaptations (choice B) will likely prove frustrating and counterproductive for Ralph. The teacher should no undervalue Ralph’s instructional needs (choice C), and avoiding decoding (choice D) would not be beneficial for Ralph. Dr. Candace Baker 12 02/01/14
  • 13. Special Education: Teaching Students with Behavioral Disorders/Emotional Disturbance Dr. Candace Baker 13 02/01/14
  • 14. Q1. A school is considering adopting a “get tough” policy that enables it to expel students whose behavior is disruptive and/or violent. Which of the following best describes the relationship between this school’s policy and IDEA? A. B. C. D. It is consistent with the requirements of IDEA. This practice probably is illegal for many students covered by IDEA. IDEA does not address administrative policies concerning students who are disruptive and/or violent. IDEA permits the expulsion of disruptive and/or violent students only if they lack the intellectual capacity to do schoolwork. Dr. Candace Baker 14 02/01/14
  • 15. Q1 -- Answer  Based on case law, the best response is B.  Under EHA (PL94-142) and its amendments, the courts have established that schools may not expel students with disabilities without due process if the behavior causing the expulsion proceedings is related to their disabilities. The provisions of EHA and IDEA that have been cited as relevant are those that call for education in the least restrictive environment and those related to placement decisions, with expulsion being viewed by the courts as an unplanned change in placement not in the student’s best educational interests. Choices A, C, and D, therefore, are incorrect. Dr. Candace Baker 15 02/01/14
  • 16. Q2. On school party days, two third-grade students with EBD get so excited that their behavior becomes very difficult to manage. The third-grade general education teacher has asked the special education teacher not to send these students to the class on the days of school parties because the students are so disruptive. Which of the following would be the best way for the special education teacher to deal with this problem? A. B. C. D. Urge the general education teacher to make allowances for these students’ behaviors in these kinds of situations. Agree to allow the students with emotional / behavioral disorders to remain in the special education classroom with a member of the nonprofessional staff during parties. Advise the students that they will not be allowed to go to school parties and offer them the option to stay home from school on those days. Help the regular education teacher arrange to have additional adults present to help with these students during parties. Dr. Candace Baker 16 02/01/14
  • 17. Q2 -- Answer choices except D, the best answer, allow the EBD students to be “excluded” from important school events with their peers. No matter how severe their behavior has been in the past, EBD students have the right to be included in current educational activities, even if that means using a variety of support strategies such as additional adult assistance. It is the EBD teacher’s role to do more than just “try to persuade” the mainstream or general education teacher to include the EBD students. It is the EBD teacher’s responsibility to see that it is done, and to help the EBD students sustain their involvement in the activity for as long as they are able.  All Dr. Candace Baker 17 02/01/14
  • 18. Q3. Which of the following is NOT an important consideration in the process of evaluating a student for evidence of EBD? A. B. C. D. The referring Teacher The age of onset of the problem behavior. The setting in which the problem behavior is exhibited The treatment to be used. Dr. Candace Baker 18 02/01/14
  • 19. Q3 -- Answer Choices A, B, and C are all considerations in the evaluation process. The correct response, D, is a part of the educational plan that is developed based on the completion of the evaluation process. Dr. Candace Baker 19 02/01/14
  • 20. Q4. If a teacher aide is assigned to assist in a classroom for students with EBD, which of the following guidelines would best help the aide to assist students who are doing independent work? A. B. C. D. Give the students frequent encouragement to get them to complete their work on their own. Provide continuous feedback to students as they work. Show students how to complete their tasks, doing some of the work for them if they delay. Limit assistance to that help needed to keep students working on their own. Dr. Candace Baker 20 02/01/14
  • 21. Q4 -- Answer Independent work periods are intended to help students learn to use their time well and strengthen their problemsolving skills. Students need to strike a balance between receiving needed assistance and coming to depend too heavily on adult intervention; thus, the best response is D. Dr. Candace Baker 21 02/01/14
  • 22. Q5. Maria, a student with EBD, has just been placed in a regular mathematics class. Two days in a row, when given a math worksheet, Maria has looked at it for a few minutes, pushed it aside, and put her head down on her desk. Which of the following does the teacher need to do before giving Maria more math worksheets in class? A. B. C. D. Ask one of the other students to work with Maria on one of the rejected worksheets. Set up a bonus point system with Maria to encourage her to do her math work. Allow Maria to take her math work home to see if she can complete it when not under pressure. Talk with Maria and evaluate whether the math assignments are appropriate for her. 22 Dr. Candace Baker 02/01/14
  • 23. Q5 -- Answer The Best Answer is D. The teacher must first determine if the math assignments are appropriate for Maria before any of the other strategies might be effective. Dr. Candace Baker 23 02/01/14
  • 24. Q6. Which of the following guidelines being stated by a teacher is an example of a dependent group contingency? A. B. C. D. “As soon as each student makes it through an entire home economics class without one reprimand for disruptive behavior, we will prepare lunch for class.” “If each student meets his or her self-monitoring goal on Tuesday, the whole class can skip a homework assignment.” Each student who finishes her or his homework during the allotted fifteen-minute homework drill will receive a coupon for a fast-food restaurant.” “Each time Johanna earns 10 points for completing academic tasks, the entire class will be rewarded with five minutes of free time.” Dr. Candace Baker 24 02/01/14
  • 25. Q6 -- Answer  Choice D, the correct response, fits the guidelines for dependent group contingencies. The peer performance of certain group members determines the consequence received by the entire group. Choices A and B represent an interdependent group contingency. In this strategy, each student must reach a prescribed level of behavior before the entire group receives a consequence. Choice C represents an independent group contingency. In this variation, the same consequence is applied to individual group members. Dr. Candace Baker 25 02/01/14
  • 26. Special Education: Application of Core Principles Across Categories of Disabilities Dr. Candace Baker 26 02/01/14
  • 27. Q1. A 13-year-old student with a severe degree of mental retardation has a verbal expressive vocabulary of a few isolated words. Which of the following augmentative communication systems can most immediately be used in a community setting with the students? A. B. C. D. Sign language Cued Speech A communication board using pictures A communication board using the alphabet and number system. Dr. Candace Baker 27 02/01/14
  • 28. Q1 -- Answer The best answer is C. A communication board using pictures is most likely to be effective given the severity of the student’s disability and the limited nature of the student’s current language skills. Dr. Candace Baker 28 02/01/14
  • 29. Q2. Computer-assisted instruction has been shown to be an important teaching method for students with mental retardation for all of the following reasons EXCEPT: A. B. C. D. The computer requires little teacher intervention, freeing the teacher to prepare upcoming lessons. The computer can be programmed to immediately deliver feedback on the correctness of a response. The computer can be programmed to provide as much repetition as a student needs. Computer graphics and sound can maintain a student’s motivation and attention to task. Dr. Candace Baker 29 02/01/14
  • 30. Q2 -- Answer correct answer is A;  You are to choose the false statement. Effective computer-assisted instruction tends to require as much teacher monitoring and intervention as other types of individualized student work.  The Dr. Candace Baker 30 02/01/14
  • 31. Q3. To best assure the transfer of word attack skills to newspaper reading, a middle school teacher of students with mild mental retardation should… A. B. C. D. prepare teacher-made newspaper articles for the students to read. select articles from the local newspaper for students to read. develop writing exercises using words from newspaper articles. prepare worksheet exercises based on single sentences from newspaper articles. Dr. Candace Baker 31 02/01/14
  • 32. Q3 -- Answer best answer is B,  Since one component of successful transfer of learning is the similarity between the situation in which a skill is learned and the situation to which it is to be applied.  The Dr. Candace Baker 32 02/01/14
  • 33. Q4. Which of the following is an example of functional language training for children who are nonverbal? A. B. C. D. Rewarding the child for making a vocalization approximating a sound made by the teacher. Rewarding the child for any vocalization made while looking at the teacher. Drilling on bilabial sounds so the student can say “mama”. Drilling the child on pointing to the sink when he or she wants a drink of water. Dr. Candace Baker 33 02/01/14
  • 34. Q4 -- Answer best answer is D, Since functional language training involves giving the child a means of influencing and interacting with the environment. Teaching the child to make sounds or imitate words in isolation does not fulfill the goal of functional language training. The Dr. Candace Baker 34 02/01/14
  • 35. Q5. A well-made teacher-developed test is generally preferred to a standardized achievement test when measuring learning mastery because it . . . A. B. C. D. is more likely to yield a true score has higher interrater reliability allows greater comparison of students to each other has better content validity Dr. Candace Baker 35 02/01/14
  • 36. Q5 -- Answer The best answer is D.  “Learning mastery” generally refers to those lessons taught in the classroom. A teacher-developed test is more likely than a standardized achievement test to assess students’ mastery of those lessons and, therefore, to have better content validity. Dr. Candace Baker 36 02/01/14
  • 37. Q6. Which of the following best describes an ecological inventory? A. B. C. D. An analysis of the curriculum of a given school system. A compilation of specific behavioral management needs of a child with disabilities A synthesis of the past educational achievements of a child with disabilities A compilation of life skills needed by a child with disabilities in present or future settings. Dr. Candace Baker 37 02/01/14
  • 38. Q6 -- Answer The best answer is D.  An ecological inventory is designed to determine those skills needed by a particular individual in his or her current and future environments. Dr. Candace Baker 38 02/01/14
  • 39. Q7. A student with behavior problems is being taught to interact appropriately with peers on the playground. The playground supervisor provides the teacher with feedback on the student’s behavior after every recess. The student earns points for acceptable behavior. When an infraction has occurred during recess, the student loses points. This contingent withdrawal of reinforcers is referred to as . . . A. B. C. D. negative reinforcement discrimination learning response cost generalization training Dr. Candace Baker 39 02/01/14
  • 40. Q7 -- Answer The correct answer is C.  Response cost refers to the removal or withdrawal of reinforcing stimuli upon the performance of an undesirable behavior. Dr. Candace Baker 40 02/01/14
  • 41. Q8. Tomas, a student in a first-grade class, is the only child standing quietly in a bus line. His teacher asks Tomas to show the other children how to stand quietly in line. Many of the students imitate Tomas’ behavior. Which of the following behavior management techniques did the teacher use? A. B. C. D. Modeling Shaping Extinction The Premack principle Dr. Candace Baker 41 02/01/14
  • 42. Q8 -- Answer The correct answer is A.  Modeling is a process by which an observer learns a response or behavior by watching and imitating the behavior of another. Dr. Candace Baker 42 02/01/14
  • 43. Q9. As mandated in IDEA, the IEP team must consider the need for transition services for each student before she or he leaves school. If it is determined that transition services are needed, which of the following represents the most critical task for the team involved in preparing the transition plan? A. B. C. D. Adopting a transition plan previously drafted by officials of the state department of education, the vocational rehabilitation agency, and the local school district. Getting information regarding post school educational services, community living options, and employment, from adult agency personnel. Providing written information for the family and student to review and suggesting a placement for postsecondary special services. Conducting meetings at which family and student goals are determined, along with personnel responsibilities, information to be collected, a time frame for providing information, and the resources needed to meet each goal. Dr. Candace Baker 43 02/01/14
  • 44. Q9 -- Answer The best answer is D.  Transition to post school life will be most effective when the goals of the student and his or her family are considered, when all responsible agencies work together to develop a plan for the student, and when specific tasks toward the goals are scheduled, monitored, and completed. Dr. Candace Baker 44 02/01/14
  • 45. Q10. For a special education teacher, which of the following is the best example of collaborative goal setting? A. B. C. D. Developing IEP goals with the regular classroom teacher and then presenting the completed goals to the students’ parents Reaching a consensus on goals by consulting with parents and the multidisciplinary team Allowing parents to choose from goals designed by the multidisciplinary team Encouraging students to select goals for IEP inclusion from teacher-approved lists. Dr. Candace Baker 45 02/01/14
  • 46. Q10 -- Answer The best answer is B,  Since collaborative goal setting requires the sharing of ideas among all those involved with the student – educators, parents, related services providers, etc. It is not appropriate in collaborative goal setting for one person or group to present previously prepared goals to the student or his or her family. There must be collaboration among all concerned to develop appropriate goals for the student. Dr. Candace Baker 46 02/01/14
  • 47. Special Education: KnowledgeBased Core Principles Dr. Candace Baker 47 02/01/14
  • 48. Q1. Which of the following techniques is likely to be most successful in helping learners with mental retardation to retain previously acquired skills? A. B. C. D. Scheduling frequent peer tutoring sessions Acknowledging appropriate behavior regularly Providing periodic review of lessons Allowing longer independent practice periods. Dr. Candace Baker 48 02/01/14
  • 49. Q1 -- Answer The best answer is C.  Once a skill has been mastered, the teacher can best assure its retention by providing additional meaningful learning situations in which the student can recall and use the skill. Dr. Candace Baker 49 02/01/14
  • 50. Q2. Which of the following is most clearly an example of a student using inappropriate syntax? A. Saying, “I see football game” B. Saying, “Wa wa” as a substitute for water C. Saying, “Me sister shoes new happy” D. Saying, “He drinked his milk” Dr. Candace Baker 50 02/01/14
  • 51. Q2 -- Answer best answer is C;  Since using appropriate syntax involves following rules for the correct arrangement of word sequences.  The Dr. Candace Baker 51 02/01/14
  • 52. Q3. Which of the following conditions is frequently attributable to Trisomy 21? A. Down syndrome B. Phenylketonuria (PKU) C. Klinefelter syndrome D. Turner syndrome Dr. Candace Baker 52 02/01/14
  • 53. Q3 -- Answer correct answer is A.  There are several possible causes of Down syndrome, with the most common being Trisomy 21, the failure of one pair of parental chromosomes to separate at conception.  The Dr. Candace Baker 53 02/01/14
  • 54. Q4. According to IDEA, when parents and schools disagree over education issues for a child with a disability, either party can request a . . . A. Mediator B. Due process hearing C. New teacher for the student D. Court date Dr. Candace Baker 54 02/01/14
  • 55. Q4 -- Answer best answer is B. According to IDEA, either a parent or The the public educational agency may initiate a hearing on the identification, evaluation, or educational placement of a student with a disability. Dr. Candace Baker 55 02/01/14
  • 56. Q5. Which of the following placement practices is required by federal law? A. Full inclusion B. Mainstreaming C. Least restrictive environment D. Resource room Dr. Candace Baker 56 02/01/14
  • 57. Q5 -- Answer correct answer is C. IDEA states that each student must be The placed in the least restrictive environment in which the unique needs of that student can be met. Choice A, B, or D may be the least restrictive environment for a given student, but that must be determined on an individual basis. Dr. Candace Baker 57 02/01/14
  • 58. Q6. According to IDEA, which of the following is a mandated responsibility of the school in servicing a student who has been identified as having a disability? A. B. C. D. Guaranteeing that the student is placed in a regular classroom in the student’s home school Paying for an independent educational evaluation of the student upon request by the parent Notifying the parent following the placement of the student in a special education program Guaranteeing appropriate access to and confidentiality of the student’s records. Dr. Candace Baker 58 02/01/14
  • 59. Q6 -- Answer The correct answer is D,  As both confidentiality of and access to students’ records are addressed in IDEA. Dr. Candace Baker 59 02/01/14
  • 60. Q7. The most useful information for developing the long-range goals and shortterm objectives on an IEP is likely to come from using the results of which of the following types of assessment? A. B. C. D. Norm-referenced tests and adaptive behavior scales Criterion-referenced tests and behavioral checklists Norm-referenced tests and language samples Developmental scales and group achievement tests Dr. Candace Baker 60 02/01/14
  • 61. Q7 -- Answer The best answer is B.  Criterion-referenced tests compare a student’s performance to a specified level of mastery or achievement. Behavioral checklists may be used to gather various types of information about the student’s current performance in a given area. Both assess specific information about a particular student, and therefore can be useful in developing IEP goals and objectives. Dr. Candace Baker 61 02/01/14
  • 62. Q8. Which of the following best describes the group of special education students for whom transition plans must be developed as part of the IEP process? A. B. C. D. Students of all ages Students between the ages of 18 and 21 Students aged 14 to 16 through school completion Students who will graduate from high school and be placed in a sheltered workshop Dr. Candace Baker 62 02/01/14
  • 63. Q8 -- Answer correct answer is C, as stated in IDEA. The Dr. Candace Baker 63 02/01/14
  • 64. Q9. An IEP must include which of the following components? A. The current level of student performance B. A record of past student performance C. A description of the student’s intellectual functioning D. Suggestions for parental involvement Dr. Candace Baker 64 02/01/14
  • 65. Q9 -- Answer correct answer is A, as stated in IDEA. The Dr. Candace Baker 65 02/01/14
  • 66. Special Education: Teaching Students with Mental Retardation Dr. Candace Baker 66 02/01/14
  • 67. 1. After scoring 100% on an oral multiplication test and repeating this performance on a similar paper-and-pencil test, a 12-year-old student with mental retardation was unable to determine the total cost of purchasing six comic books when the price was $3 for one comic book. This student is most clearly deficient in which of the following areas? A. Memory skills B. Computational skills C. Conceptual skills C. Generalization skills Dr. Candace Baker 67 02/01/14
  • 68. Q1 -- Answer best answer is D. Poor generalization skills are exhibited when an individual cannot use a learned skill under conditions different from the learning situation. The Dr. Candace Baker 68 02/01/14
  • 69. Q2. Which of the following statements best explains the increase in the number of children identified as having mild mental retardation at the time this population reaches school age? A. B. C. D. Children with mental retardation tend to exhibit a cognitive slump at about age 6. Mental retardation is easier to detect at this age because emphasis is placed on academic achievement. A label of mild mental retardation cannot legally be given to children before they reach school age. School personnel are generally more eager than parents to find the reasons for children’s learning difficulties. Dr. Candace Baker 69 02/01/14
  • 70. Q2 -- Answer The best answer is B.  Young children with mild mental retardation often have age-appropriate adaptive behavior and social skills, and therefore perform adequately in the nonacademic situations encountered before school begins. They begin to exhibit problems, however, when the academic demands of the classroom are placed on them. Dr. Candace Baker 70 02/01/14
  • 71. Q3. According to the definition used by the American Association on Mental Retardation (AAMR), which of the following is one of the conditions required before an individual can be said to have mental retardation? A. B. C. D. An IQ score below 50 A known cause of the individual’s condition Deficits in adaptive behavior Deficits in social skills Dr. Candace Baker 71 02/01/14
  • 72. Q3 -- Answer the choices given, C is the only condition specified in the AAMR definition of mental retardation. Of Dr. Candace Baker 72 02/01/14
  • 73. Q4. The most recent conceptual thinking related to program delivery options for individuals with mental retardation gives the greatest amount of emphasis to the… A. B. C. D. development of skills that will promote independence in an institutional setting development of skills that will help with integration into the community mastery of academic skills mastery of computer technology Dr. Candace Baker 73 02/01/14
  • 74. Q4 -- Answer best answer is B. Although the process of successful integration into the community is extremely complex both conceptually and practically, it has been noted by many educators, researchers, and advocacy groups in recent years that it is essential for appropriate programming to occur in school to assure such success for individuals with mental retardation. The Dr. Candace Baker 74 02/01/14
  • 75. Q5. Using a functional approach is important when designing instruction for students with moderate and severe degrees of mental retardation because this approach… A. B. C. D. follows a sequence according to normal child developmental milestones breaks down the skills to be taught into a series of discrete behaviors and then sequences those behaviors into the progression in which they occur when performing the skill concentrates on behaviors that are critical to performing a necessary task teaches those skills that occur naturally within a school environment. Dr. Candace Baker 75 02/01/14
  • 76. Q5 -- Answer correct answer is C, since the functional approach emphasizes teaching skills that the students need now or will need in the future. The Dr. Candace Baker 76 02/01/14
  • 77. Q6. Which of the following procedures is likely to be most effective in teaching basic arithmetic concepts to students with mental retardation? A. Using manipulatives and pictures for counting and grouping B. Completing paper-and-pencil drilland-practice worksheets C. Using calculators for problem-solving D. Using flash cards for learning basic addition and subtraction facts. Dr. Candace Baker 77 02/01/14
  • 78. Q6 -- Answer  The best answer is A, since counting and grouping are two of the basic elements required in learning arithmetic concepts. All children who are learning basic arithmetic concepts are assisted in this effort if manipulatives, pictures, or other concrete representations of the concepts are presented. Dr. Candace Baker 78 02/01/14
  • 79. Professional Assessments for Beginning Teachers Special Education 79 (0381) Text Questions & Answers 02/01/14
  • 80. Q1. Which of the following is the focus of the noncategorical or cross-categorical approach to special education? A. B. C. D. E. The use of self-contained special education classes for students with disabilities The differentiation of students’ needs based on their specific disabilities The education of students with disabilities based on their similar characteristics and needs The use of differential diagnosis as a technique for distinguishing among disabilities The application of task-analysis techniques in the instruction of students with severe disabilities. Dr. Candace Baker 80 02/01/14
  • 81. Q1 -- Answer The best answer is C.  Schools or districts that use a noncategorical or cross-categorical approach to special education generally base that decision on a belief that the etiology of students’ disabilities is not important. Rather, the students’ instructional needs should be assessed, and students with similar needs should receive similar treatment. Dr. Candace Baker 81 02/01/14
  • 82. Q2. Which of the following is a factor that impedes researchers’ attempts to determine with certainty the prevalence of learning disabilities in the United States? A. B. C. D. E. The absence of any definition of learning disabilities in federal guidelines for special education A general reluctance on the part of educators and diagnosticians to classify students as having learning disabilities The overlap in identification criteria between the classifications of severe learning disabilities and severe emotional disturbance The tendency of school districts to place students identified as having learning disabilities in regular rather than special education classes The lack of a precisely defined cut-off point at which a learning problem requiring remediation becomes a disability requiring special education. 82 Dr. Candace Baker 02/01/14
  • 83. Q2 -- Answer  The best answer is E.  Definitions of learning disabilities are often imprecise in their guidance regarding the point at which a specific learning problem should be classified as a disability, a factor that affects identification of the condition. Choice A is incorrect because the Individuals with Disabilities Education Act defines specific learning disabilities in general terms. B is incorrect; the numbers indicate just the opposite. C is incorrect because identification criteria for these two areas do not present an appreciable problem of overlap. D is irrelevant; placement is not the issue of the question. Dr. Candace Baker 83 02/01/14
  • 84. Q3. The total number of children with disabilities who are served in special education programs has grown at a phenomenal rate since 1975. Which of the following is the most likely reason for the increase? A. B. C. D. E. The passage of PL 94-142, which required that the public schools educate all children with disabilities from age three through twenty-one Improved assessment techniques that made it easier to locate and identify children with disabilities An increase in the number of special education teachers trained to serve students with disabilities A general decrease in school enrollment resulting in more space for special education students An increase in federal aid for education Dr. Candace Baker 84 02/01/14
  • 85. Q3 -- Answer The best answer is A.  The Education for All Handicapped Children Act (PL 94-142), passed in 1975, was the first law to mandate a free and appropriate public education for all children and youth with disabilities. Prior to that time, schools could exclude children they felt were not educable. Dr. Candace Baker 85 02/01/14
  • 86. Q4. Kevin is a second grader whose gradeequivalent score on a reading test is 5.0. Which of the following is an accurate interpretation of Kevin’s score? A. B. C. D. E. Kevin has the reading skills of the average fifth grader. Kevin can read fifth-grade materials. Kevin correctly answered as many questions on the test as did the average beginning fifth grader. Kevin has progressed three years beyond the level at which he is expected to be reading. Kevin has mastered the skills of the reading curriculum up to the fifth-grade level. Dr. Candace Baker 86 02/01/14
  • 87. Q4 -- Answer The best answer is C. A grade-equivalent score for an individual represents the score on the test that is the same as the median, or average, score for all students at that grade level in the norming group. Therefore, if Kevin obtained a grade-equivalent score of 5.0 on a reading test, he correctly answered the same number of questions on the test as did the average beginning fifth grader. Dr. Candace Baker 87 02/01/14
  • 88. Q5. Of the following methods of assessment, which is likely to provide the LEAST useful information for instructional planning? A. Intelligence tests B. Behavioral checklists C. Criterion-referenced achievement tests D. Systematic observations E. Informal skill inventories Dr. Candace Baker 88 02/01/14
  • 89. Q5 -- Answer The best answer is A.  It has been widely acknowledged in the field of education that, although intelligence tests may serve several appropriate purposes, they provide teachers with limited useful information. Results of intelligence tests are unlikely to help teachers decide what to teach or how to teach it. Dr. Candace Baker 89 02/01/14
  • 90. Questions 6 – 8 are based on the following information.  Michelle, a 19-year-old whose parents are deceased, has profound mental retardation and lives in a group home. She is bused to a special class by the local school district. Michelle has had frequent, extended absences from school because of her frail physical condition. She is nonambulatory and nonverbal; her vision and hearing are intact. She can communicate a few basic needs by means of a pictorial language board. Dr. Candace Baker 90 02/01/14
  • 91. Q6. Which of the following is the appropriate focus of Michelle’s individualized education program (IEP)? A. Functional academics B. Basic academic skills C. Prevocational skills D. Self-care skills E. Oral language Dr. Candace Baker 91 02/01/14
  • 92. Q6 -- Answer The best answer is D.  Because of Michelle’s profound level of mental retardation and the severe nature of her secondary disabilities, choices A, B, C, and E are not likely to be realistic or useful areas on which to focus. Self-care skills, however, would help Michelle function better both at school and at home. Dr. Candace Baker 92 02/01/14
  • 93. Q7. Michelle’s IEP would be expected to include goals and objectives in all of the following EXCEPT . . . A. Recreation B. Articulation C. Feeding D. Toileting E. Communication Dr. Candace Baker 93 02/01/14
  • 94. Q7 -- Answer The best answer is B.  Michelle, at age nineteen, is still nonverbal. Therefore, articulation is an inappropriate area for which to include goals and objectives on her IEP. Dr. Candace Baker 94 02/01/14
  • 95. Q8. Since Michelle’s parents are deceased, who is required to review and approve her educational program? A. B. C. D. E. A representative of the state agency for children’s services A court-designated staff member of the group home in which she resides A blood relative willing to assume this responsibility A surrogate parent appointed by the local education agency A member of the local school board where Michelle attends school Dr. Candace Baker 95 02/01/14
  • 96. Q8 -- Answer best answer is D, as stated in PL 94-142 and subsequent legislation. The Dr. Candace Baker 96 02/01/14
  • 97. Q9. Which of the following is most important for teachers of students with disabilities to do during scheduled conferences with parents? A. B. C. D. E. Use professional terminology when referring to the student’s disability and instructional needs Express agreement with the parents’ opinions about their child’s strengths and weaknesses Use the time in a leisurely, unstructured manner so that issues and concerns will arise spontaneously Determine whether the parents have hidden feelings of frustration regarding their child’s disability Present work samples and other evidences of the student’s skills and discuss ideas for future activities. Dr. Candace Baker 97 02/01/14
  • 98. Q9 -- Answer  The best answer is E.  Student work samples and other evidence are useful for giving parents a concrete representation of their child’s progress, and teachers should include parents in a discussion of educational planning. A would be inappropriate. B is not an especially helpful policy in itself and would be dishonest if agreement did not exist. C is incorrect; a conference should be wellprepared so the time can be used productively. D would be inappropriate. Dr. Candace Baker 98 02/01/14
  • 99. Q10. A first-grade teacher has established a system in which students earn points by complying with classroom rules and completing work. The teacher also takes points away fr0m students who do not follow rules and do not complete work. When taking away points, the teacher is using which of the following techniques? A. B. C. D. E. Negative reinforcement Overcorrection Response cost Timeout Differential reinforcement Dr. Candace Baker 99 02/01/14
  • 100. Q10 -- Answer The best answer is C.  Response cost is a behavior-management strategy in which a positive reinforcing stimulus (in this case, earned points) is removed following the performance of an undesirable behavior. Dr. Candace Baker 100 02/01/14
  • 101. Educable Mentally Handicapped 101 (0381) Text Questions & Answers 02/01/14
  • 102. Q1. In relation to cognitive development, most five-year-old children are able to: A. B. C. D. Use numerical concepts to solve problems. Identify abstract similarities and differences. Draw conclusions based on their own experience. Generalize rules from one situation to another. Dr. Candace Baker 102 02/01/14
  • 103. Q1 -- Answer  The  best answer is C. In terms of their cognitive development, five-year-old children typically exhibit the ability to draw conclusions based on their own experience. The fact that their thinking is often characterized by a number of faulty assumptions and conclusions is more a result of their extremely limited experience than an inability to draw what are to them reasonable conclusions. On the other hand, most five-yearolds are unable to use numerical concepts to solve problems (choice A), identify abstract similarities and differences (choice B), or understand how rules can be generalized from one situation to another (choice D). These cognitive abilities are generally developed by children during middle and later childhood. Dr. Candace Baker 103 02/01/14
  • 104. Q2. Piaget’s Theory is based on the premise that children’s cognitive development depends primarily upon: A. B. C. D. The appropriate use of positive and negative reinforcers by parents and other caretakers. Their unconscious drives and motives. Their attainment of a biologically based, sequential series of developmental stages. The availability of appropriate role models. Dr. Candace Baker 104 02/01/14
  • 105. Q2 -- Answer  The best answer is C.  This question requires knowledge of a major theory of human cognitive development. According to Piagetian theory, intellectual development occurs in a universal sequence of stages, each stage building on cognitive skills attained during previous stages and involving progressively more complex types of thought. The universality of the developmental stages observed by Piaget in children worldwide caused him to hypothesize further that these stages reflect an immutable and biologically based element common to all human populations (choice C). The alternative choices listed all characterize other well-known theories of development. The use of positive and negative reinforcers (choice A) as primary external factors in cognitive development characterizes behaviorist theories of development rather than Piaget’s theory. A child’s unconscious drives and motives (choice B) are the underlying factors in a variety of psychoanalytic approaches that focus on the subconscious as a primary determinant of human development. Finally, the availability of appropriate role models (choice D) is viewed as an important causal factor in social learning theory but not in Piagetian theory. Dr. Candace Baker 105 02/01/14
  • 106. Q3. Which of the following cognitive disabilities is caused by the presence of an extra chromosome? A. B. C. D. Tay-Sachs disease Fragile X Down syndrome Phenylketonuria (PKU) Dr. Candace Baker 106 02/01/14
  • 107. Q3 -- Answer  The  best answer is C. Down syndrome is a cognitive disability caused by a genetic abnormality involving the presence of an extra twenty-first chromosome (choice C). Although the other conditions listed in the question are also of genetic origin, none is caused by the presence of an extra chromosome. Tay-Sachs disease (choice A) is the result of a recessive gene. It causes rapid mental and physical deterioration after birth and typically leads to death during early childhood. Fragile X (choice B) involves an abnormality of the X chromosome that results in mental impairment, particularly in males. Phenylketonuria (PKU) (choice D) is caused by a recessive gene that affects phenylalnin metabolism. If left untreated, PKU can also cause a severe cognitive disability. 107 Dr. Candace Baker 02/01/14
  • 108. Q4. Mr. Bronson’s sixth grade class includes students from several ethnic minority groups. To plan an effective classroom management strategy, it is important for Mr. Bronson to recognize that these students: A. B. C. D. Generally wish to assimilate into the dominant culture. Are likely to be openly hostile to a white male teacher. Will see one another as allies in overcoming discrimination. May form ethnic peer groups that become isolated from another. Dr. Candace Baker 108 02/01/14
  • 109. Q4 -- Answer The best answer is D.  In typical social development, preadolescent children are likely to form social groups based on perceived similarity (choice D). A teacher wishing to promote ethnic diversity cannot assume that members of various ethnic groups wish to assimilate into the dominant culture (choice A). Students will respond to the teacher on the basis of their previous school and family experiences and will not necessarily be hostile to a whit male teacher (choice B). Because children of this age group tend to form social groups based on obvious similarities, they will not necessarily see one another as allies (choice C) unless they are provided with instruction and structured experiences that help them understand discrimination. Dr. Candace Baker 109 02/01/14